El fotón y la masa


Pedazo de calabaza.

El tema de la masa del fotón es muy controvertido. Es evidente que es difícil de tragar eso de que los fotones no tienen masa. En nuestra vida diaria estamos acostumbrados a que todo lo que nos rodea tiene masa y aceptar que haya algo sin ella no es trivial del todo.

¿Pero por qué narices se empeñan los físicos en repetir que el fotón no tiene masa?

¿Acaso el fotón no tiene energía? ¿Entonces no es válida la relación E=mc^2?

Estas son preguntas que se repiten una y otra vez y que son ciertamente complicadas de responder. Esta entrada, que se preveé árida, intentará dar los argumentos teóricos existentes para mostrar el por qué se dice que el fotón es una partícula sin masa.

El argumento relativista

En relatividad especial trabajamos en un espaciotiempo que tiene una métrica de Minkowski. En dicha teoría, el módulo de cualquier vector (de cuatro coordenadas y calculado con dicha métrica) tiene que ser el mismo para todo observador inercial (se mueve en línea recta con una velocidad constante en módulo).

Para repasar algo de relatividad especial dejamos aquí dos entradas, la primera más suave y la segunda más técnica:

Conceptos de Relatividad Especial

Revisión de Relatividad Especial 1

Espacio de Minkowski, vectores y módulos

El espacio de Minkowski es un espacio de 4 dimensiones, el tiempo y las tres espaciales, que representa nuestro espaciotiempo. Además, introduce una forma curiosa de calcular los módulos de los vectores que se pueden definir en dicho espacio empleando la llamada métrica de Minkowski.

Como no nos es posible dibujar cuatro dimensiones nos restringiremos a dos, así en vez de trabajar con vectores con componentes (ct, x, y, z) trabajaremos solo con (ct, x). Esto no le quita generalidad a la discusión ya que la extensión es directa.  Notemos que en la componente temporal introducimos el factor c que representa la velocidad de la luz, esto lo hacemos para que las unidades de todas las coordenadas sean la misma. Si el tiempo se mide en segundos (s) y c en metros partido por segundo (m/s) es evidente que el producto ct se mide en metros.

Espacio de Minkowski con la línea que representa la trayectoria espaciotemporal de un rayo de luz o fotón.

Una partícula que se mueva a la velocidad de la luz verificará por tanto que el cociente entre el espacio recorrido y el tiempo que tarda en recorrerlo será c.

\dfrac{x}{t}=c

Por lo tanto,

\dfrac{x}{ct}=1

Y por esto, las partículas que se mueven a la velocidad de la luz se dibujan formando un ángulo de 45º con los ejes de coordenadas.

Ahora bien, lo que implica esto es que tenemos para partículas que se mueven a la velocidad de la luz lo siguiente:

x=ct

Minkowski nos dice que en su espacio si queremos calcular el módulo de un vector (t,x) lo tenemos que hacer de la siguiente forma:

s^2=(ct)^2-x^2

Esto contrasta con lo que nos enseñan en el colegio de que el módulo del vector dado por (x,y) se calcula como d^2=x^2+y^2.

Así, si tenemos una partícula que se mueve a la velocidad de la luz se verifica que x=ct y por tanto s^2=0.  La relatividad especial nos dice que cualquier observador inercial ha de estar de acuerdo con este módulo, es decir, esta cantidad es un invariante físico. Si cambiamos de coordenadas, por cambiar de observador inercial, variaremos los valores de x y ct a x’ y ct’.  Pero el cambio es de tal forma que se compensan a la hora de calcular los módulos.

Sabemos que los tiempos se dilatan y las longitudes se contraen cuando un observador compara las medidas de tiempos y longitudes con las de otro observador distinto, y lo hacen justo así para que los módulos de los vectores en cuatro dimensiones permanezcan inalterados.

Si uno quiere calcular el momento de una partícula en cuatro dimensiones utilizará la expresión P=(\dfrac{E}{c},p), donde la primera componente nos da la energía y la segunda componente el momento de la partícula.

Si calculamos el módulo de este vector para partículas que se mueven a la velocidad de la luz obtendremos:

\left(\dfrac{E}{c}\right)^2-p^2=0

E^2=p^2c^2

E=pc

Lo que quiere decir es que toda su energía procede del hecho de que tiene momento, es decir, movimiento.

Todos los 4-vectores que se definen en el cono de luz, en la superficie que está formando 45º con los ejes (el cono de luz), vienen definidos por la propiedad de que su módulo es nulo. A estos vecotres se los denomina nulos por razones obvias. Y evidentemente, una partícula que se mueva a la velocidad de la luz no puede variar dicha velocidad en ningún momento, ha de permanecer en el cono.

Para partículas que se mueven a velocidades inferiores a la velocidad de la luz lo que obtenemos es:

\left(\dfrac{E}{c}\right)^2-p^2=m^2c^2

E^2=p^2c^2+m^2c^4

Donde esa m es la masa de la partícula medida en reposo.  Evidentemente, si la partícula está en reposo su momento p=0, y por tanto:

E^2=m^2c^4

E=mc^2

Y vemos aquí por qué esta expresión SOLO ES VÁLIDA PARA PARTÍCULAS EN REPOSO. Un fotón, se mueve siempre a la velocidad de la luz, toda su energía viene de su momento (de estar en movimiento) y no podemos pararlo (en el vacío c siempre ha de ser c, si intentamos frenar un fotón este desaparece). Por lo tanto no podemos decir que como un fotón tiene energía entonces tiene masa. Lo que podemos decir a la vista de la expresión:

E^2=p^2c^2

Es que un fotón no tiene masa y como esto procede de un módulo de un vector en 4 dimensiones, cualquier observador inercial diría lo mismo.

El argumento gauge

Las simetrías gauge son las transformaciones que podemos hacer sobre los objetos matemáticos de nuestras teorías de forma que todos los resultados observables permanezcan inalterados.

Si queréis una lectura sobre qué es eso de simetría gauge para aclarar ideas:

Gauge esto, gauge lo otro… ¿Qué es una teoría gauge?

Las teorías gauge son interesantes por dos motivos:

  1. Involucran cantidades conservadas (en su versión global).
  2. Determinan la forma de las interacciones físicas (en su versión local).

Un ejemplo de andar por casa

La mayoría nos hemos peleado con los problemas del instituto:

Sea un cuerpo de masa m a una altura h del suelo que dejamos caer. ¿Con qué velocidad llegará al suelo?

Estos problemas ser resolvían fácilmente aplicando la conservación de la energía.

E_p^{inicial}-E_p^{final}=E_c^{final}-E_c^{inicial}

La E_p representa la energía potencial gravitatoria (mgh) y la E_c la energía cinética ((1/2)mv^2).

¿Qué pasa si yo decido sumar a todas mis energías potenciales una cantidad constante K?  Entonces, tendríamos lo siguiente:

(E_p^{inicial}+K)-(E_p^{final}+K)=E_c^{final}-E_c^{inicial}

Pero, evidentemente, eso queda exactamente igual que si no hubieramos sumado nada:

(E_p^{inicial}+K)-(E_p^{final}+K)=E_p^{inicial}+K-E_p^{final}-K=E_p^{inicial}-E_p^{final}

Este hecho nos permitía poner el origen de potencial donde queríamos según nos conviniera mejor a la hora de resolver el problema, bastaba con restar o sumar una cantidad fija a todas las energías potenciales para tener el valor 0 donde quisieramos. Y eso, no cambiaba la física que estábamos estudiando.

Gauge

Una cosa análoga se puede hacer con el electromagnetismo. Sabemos que el campo electromagnético está definido por campos eléctricos \vec{E}=(E_x,E_y,E_z) y campos magnéticos \vec{B}=(B_x,B_y,B_z). Como los físicos son muy vagos en vez de trabajar escribiendo tantas componentes diseñan una tabla:

A este bicho F_{\alpha\beta} lo llamamos tensor electromagnético y solo es una forma compacta de tener todas las componentes del campo electromagnético ordenaditas para facilitar su manejo.

Seguro que alguna vez nos hemos topado con el concepto de potenciales. El potencial gravitatorio es la energía potencial gravitatoria por unidad de masa, conocido dicho potencial en todos sus puntos podemos calcular el campo gravitaorio. También sucede esto con el campo electromagnético. De hecho, se puede definir un objeto, denominado 4-potencial electromagnético:

A_{\alpha}=(\phi/c,A_x, A_y, A_z)

donde la primera componente nos da el potencial eléctrico, con el que calcularíamos el campo eléctrico.  Y las tres restantes forman lo que se conoce como el potencial vector, con lo que se calcula el campo magnético.

Lo interesante de esto es que se verifica:

F_{\alpha\beta}=\partial_\alpha A_\beta-\partial_\beta A_\alpha

Esta expresión lo único que nos dice es que podemos calcular todas las componentes del tensor electromagnético (campos eléctricos y magnéticos) tomando derivadas (respecto de las cuatro coordenadas (t,x,y,z)) del 4-potencial electromagnético.

Ahora viene lo chulo, ¿qué pasa si yo cambio el 4-potencial de la siguiente manera?

A_\alpha+\chi

Es decir, le sumamos un factor \chi. Por cuestiones técnicas, que vendrán al caso en un momento, este factor se considera que es la derivada de una función escalar f.

A_\alpha+\partial_\alpha f

¿Qué le ocurre al tensor electromagnético si cambio esto?

F'_{\alpha\beta}=\partial_\alpha (A_\beta+\partial_\beta f)-\partial_\beta (A_\alpha+\partial_\alpha f)

F'_{\alpha\beta}=\partial_\alpha A_\beta+\partial_\alpha\partial_\beta f - \partial_\beta A_\alpha-\partial_\beta\partial_\alpha f

F'_{\alpha\beta}=\partial_\alpha A_\beta-\partial_\beta A_\alpha+(\partial_\alpha\partial_\beta f-\partial_\beta\partial_\alpha f)

En principio no parece que nos quede igual que al principio y eso sería malo porque si cambiamos el tensor electromagnético entonces cambiaríamos la física. Pero resulta que matemáticamente sabemos que para funciones escalares (bien comportadas y tal, ya sabéis todo eso que los matemáticos dicen y que aquí se cumple) las derivadas cruzadas son iguales independientemente del orden en el que se hagan, por ejemplo:

\dfrac{\partial}{\partial x}\dfrac{\partial}{\partial y}f(x,y)=\dfrac{\partial}{\partial y}\dfrac{\partial}{\partial x}f(x,y)

Es decir, que:

\partial_\alpha\partial_\beta f-\partial_\beta\partial_\alpha f=0

Y por tanto:

F'_{\alpha\beta}=\partial_\alpha A_\beta-\partial_\beta A_\alpha=F_{\alpha\beta}

¿Y esto por qué es importante?

Esto es importante por lo siguiente:

  • Los físicos no solon quieren tener tablas con la información de los campos físicos. También quieren saber como se comportan dichos campos. En el caso anterior no solo queremos definir el tensor electromagnético F_{\alpha\beta}, también queremos obtener las ecuaciones que verifica, en este caso las ecuaciones de Maxwell.
  • Para conseguir esto se define un objeto que se llama lagrangiana. A partir de ella, tomando diferentes derivadas, se obtienen las ecuaciones físicas de los campos.
  • En el caso electromagnético, el lagrangiana tiene el siguiente aspecto:

L=-\dfrac{1}{4}F_{\alpha\beta}F^{\alpha\beta}

  • Recordemos que el tensor electromagnético, como hemos visto antes, se puede expresar como combinaciones de derivadas del 4-potencial A_\alpha. Este es el objeto físico relevante en la lagrangiana.
  • Si los fotones tuvieran masa aparecería explícitamente un término cuadrático (producto de dos) en los 4-potenciales:

L=-\dfrac{1}{4}F_{\alpha\beta}F^{\alpha\beta}+\dfrac{1}{2}m_{foton}^2A_\alpha A^\alpha

  • Pero si cambiamos el 4-potencial como hemos hecho antes, que hemos visto que el tensor electromagnético no cambia, pasa lo siguiente:

L=-\dfrac{1}{4}F_{\alpha\beta}F^{\alpha\beta}+\dfrac{1}{2}m_{foton}^2(A_\alpha +\partial_\alpha f)^2

Si desarrollamos ese cuadrado tendremos que la lagrangiana adquiere nuevos términos. Eso implica que las ecuaciones del electromagnetismo variarían para cada elección de función f y eso no puede ser porque la física no puede ser sensible a la elección de dicha función si queremos que todo funcione como hasta ahora.

Así que si la invariancia gauge es acertada, y tiene que serlo porque determina la conservación de la carga eléctrica y la interacción electromagnética que conocemos, la masa del fotón tiene que ser nula.

Otros argumentos

Polarización de la luz

La luz tiene una característica y es que puede ser polarizada. Para refrescar este concepto:

Polarización ¡que sustos nos das!

Planck, no compres sin Thom ni son…

La luz es una onda, oscila y se propaga, pero lo puede hacer en diferentes direcciones. Polarizar la luz, groso modo,  significa seleccionar un plano de oscilación.

Sea del modo que sea, la polarización de la luz se puede describir con solo dos datos. Podemos tener polarización lineal, circular o elíptica. En el caso de la polarización lineal los datos serán las componentes vertical y horizontal de la luz polarizada, en el caso de polarización circular o elíptica si está girando a derecha o a izquierda:

Si la luz estuviera compuesta por partículas con masa no bastarían dos datos sino que necesitaríamos tres para describir los posibles estados de polarización. Así que una polarización del tipo que sufre la luz solo es compatible si está formada por partículas sin masa.

Alcance de la interacción electromagnética

Hasta la fecha, que yo sepa, no hay pruebas de que el alcance de la interacción electromagnética sea finito. Es decir, la interacción se extiende desde una carga hasta el infinito.

Eso, en física elemental, se traduce en el hecho de que el potencial eléctrico sea de la forma:

V=K\dfrac{q}{r}

Si la interacción tuviera un alcance finito, como es el caso de la interacción débil, la forma de este potencial sería:

V=K\dfrac{q}{r}e^{-\dfrac{m_{foton}c}{\hbar}r}

Esto evidentemente, induciría cambios en la ley de Coulomb. Sin embargo, no hay evidencias experimentales de este hecho, por lo tanto, hasta ahora el fotón no tiene masa.

Ese potencial es el que describe la interacción nuclear fuerte en los núcleos que está mediada por partículas llamadas piones que tiene mucha masa, dicha interacción tiene un alcance muy reducido, justamente debido a la masa de sus mediadores.

Conclusión

Hay muchas formas de llegar a la conclusión de que el fotón es una partícula sin masa. Aquí hemos expuesto algunas que espero que hayan aclarado el tema. Haremos un resumen para fijar ideas, o por si has saltado hasta aquí 🙂 :

  1. Una partícula que se mueve a la velocidad de la luz en el vacío no puede tener masa en resposo.
  2. El fotón aparece como el mediador de la interacción electromagnética. Esta interacción es invariante bajo una transformación gauge. Esto implica que se conserva la carga eléctrica y que el electromagnetismo tiene un determinado comportamiento físico. La invariancia gauge, además, prohibe que las partículas que median la interacción relacionada tengan masa.
  3. Se pueden buscar pruebas más «evidentes» como que la luz solo tiene dos dimensiones o que el potencial eléctrico es de tipo Coulombiano. Si el fotón tuviera masa estos hechos serían diferentes.

Podemos cerrar diciendo que, hasta lo que sé, no hay evidencias experimentales de que el fotón tenga masa en reposo. El problema es que si miramos las tablas de partículas no dirán que la masa del fotón estará por debajo de 10^{-15}-10^{-26} eV. No nos dirán que es cero, simplemente porque demostrar que algo es justamente 0 es ciertamente difícil, los experimentadores están condenados por la sensibilidad de sus aparatos y apreciar por debajo de eso no es posible. (Pero vamos, que para mí esas cotas huelen a cerete).

Datos de cotas de masa del fotón a 2012. Particle Data Group.

Referencias

Photon and Graviton Mass Limits

Un review muy completo de los modelos teóricos y de los experimentos realizados sobre este tema. Actualizado el 5 de Octubre del 2010.

Si conocéis fuentes más actuales serán bienvenidas.

Nos seguimos leyendo…

131 Respuestas a “El fotón y la masa

  1. Uno de los problemas no definidos y que tiene incidencia en la discusión es el concepto de materia. Este puede expresarse de la siguiente forma: la Masa en reposo es la medida de la cantidad de materia.
    El fotón tiene masa o información cuantitativa de masa pero no materia.
    Saludos

  2. Iñigo C. Sánchez

    ¿Y si pensaramos que la luz como energía arrancaría masa de allí donde pasara, de ahí su comportamiento como energía y masa al mismo tiempo en el experimento de la rejilla? ¿Y siguiendo con esta idea loca, al momento de poseer masa se pudiera acelerar con masas muy grandes como pudiera ser un agujero negro, actuando éste como un acelerador?

  3. «Donde esa m es la masa de la partícula medida en reposo. Evidentemente, si la partícula está en reposo su momento p=0, y por tanto:

    E^2=m^2c^4

    E=mc^2

    Y vemos aquí por qué esta expresión SOLO ES VÁLIDA PARA PARTÍCULAS EN REPOSO».

    Primero: E=mc^2 es una igualdad, de donde se deduce que las partículas en reposo pueden transformarse en energía y viceversa, que la energía puede trasnformarse en partículas en reposo.

    Segundo: el que las partículas en reposo se transformen en energía no quita para que dicha energía esté compuesta de fotones.

    Tercero: lo que quiere decir que existe algún proceso que desconocemos mediante el cual los fotones adquirien masa y se transforman en partículas.

    Y cuarto: para no entrar en contradicción con la física… podriamos considerar a la masa no como materia, sino como energía condensada o solidificada?

  4. Héctor Da Silva

    La fórmula correcta que permite obtener el límite de masa del fotón es
    limite de masa = 2h/ long de onda de Compton reducida.C
    El valor coincide muy bien con los valores publicados en la página La mula de Francis de Francis Naukas sobre la masa del foton y del graviton.
    Y oh! sorpresa coincide «bastante bien» con la masa del gravitón.
    Si se verificara que el valor es el correcto para el Gravitón, podríamos estar a un paso de obtener el valor adecuado para la constante cosmológica y estaríamos a un paso de develar el secreto de la materia oscura.
    Saludos.

  5. Héctor Da Silva

    He escrito mal las fórmulas anteriores, es
    límite de masa = h/long onda compton reducida. C = h . 2pi / long de onda de Compton .C = h reducida.4 pi^2/long onda de Compton
    Esta ecuación describe con mucha aproximación los límites de masa establecidos experimentalmente en la página mencionada anteriormente en experiencias gravitatorias, usando la ley de Coulomb, etc, y su valor se aproxima a los valores de masa del gravitón con menor exactitud.

  6. Cotas experimentales a la masa del fotón:

    Haz clic para acceder a Foton.pdf

  7. Héctor Da Silva

    Los límites de masa del fotón y graviton según la página http://arxiv.org/pdf/0809.1003.pdf. responden a la siguiente ecuación:

    M = h reducida / long de onda de Compton . C = h / long onda de Compton reducida . C

  8. Héctor Da Silva

    Si consideramos que el fotón es una partícula másica de energía cinética E = 1/2 MC² y esta es igual a la energía del fotón o quanto, que es E = h f = h C /long onda, igualando tendremos M = 2 h / C . long onda.
    Si consideramos al fotón como una dualidad onda partícula, De Broglie estableció que para una partícula masica existía una onda asociada cuya energía era E = long de onda / p.
    Teniendo en cuenta que a la velocidad de la luz E/p = C tendremos para el fotón másico, cuya energía es E = 1/2 M C², una p =1/2 M C.
    Al igualar tendremos que M = 2h/C . long de onda, el mismo resultado anterior.
    Utilizando este valor de masa del foton para calcular el radio de una órbita a velocidad C alrededor del agujero negro, mediante consideraciones Newtonianas, obtendremos el radio de Schuwarlszchild, calculado, generalmente mediante consideraciones relativistas.

    • Manuel Sanchez Carrilero

      Para empezar la energía cinética del fotón no es (1/2)Mc^2 ya que esta fórmula corresponde a la expresión clásica de energía cinética que no viene al caso.

      En segundo lugar nunca podríamos considerar al fotón como una partícula másica ya que su masa es siempre CERO.

      Finalmente lo que afirmas que dijo De Broglie no es correcto: «estableció que para una partícula másica existía una onda asociada cuya energía era E = long de onda / p.»

      La única energía que existe, en estos casos de cuantización, cumple con la ecuación de Planck, E = hf, donde f es la frecuencia y h una constante.

      Además lo que realmente dijo De Broglie es que la longitud de onda asociada es h/p = h/(mv), esto para el caso de una partícula clásica; para el caso de una partícula relativista de masa en reposo mo, pero con una velocidad v < c, la expresión para la longitud de onda asociada sería:

      L = h/(mR*v)

      donde mR es un factor mal llamado algunas veces "masa relativista" dado por la expresión

      mR = mo/SQR(1-(v/c)^2)

      Vemos que la longitud de onda disminuye asintóticamente con la velocidad careciendo de todo sentido para el caso del fotón en que la masa es mo = 0 y con una velocidad v = c.

      Te sugiero que te leas mi comentario aquí mismo de fecha 23ENE19 en que he redactado algo que pienso puede ser de utilidad.

      • Héctor Da Silva

        Es cierto, me equivoqué al colocar la fórmula de De Broglie, pero haciendo la correspondiente corrección de su expresión el resto es correcto y la con lusión está bien, solo cometí el error al escribirla. Si se puede editar la respuesta ya lo corrijo.
        Con respecto a las otras objeciones son comunes a los que conocen y siguen la teoría de la relatividad, no son nuevas, y me la han repetido una y otra vez dando a conocer mis «errores», por ejemplo que consideran que el fotón no tiene masa, cuando eso es lo que cuestiono con mi planteo.
        Con respecto a la ecuación clásica de la energía cinética uso E = 1/2MV², Considerando que M es la masa real a cualquier velocidad V. En este caso no he utilizado la ecuación E = 1/2 M0 V² ya que M0, es masa en reposo (a altas velocidades corregida por la velocidad) y el fotón no tiene masa en reposo M0, por lo tanto siempre obtendré E = 0.
        Por lo tanto utilizé para Energía Cinética E = 1/2 MC².
        En cierta forma el fotón es determinista y Newtoniano.

        Originalmente obtuve el valor de Masa =
        2 h/long de onda .C y lo chequee con los valores de límite de masa del fotón publicados en
        Masa del fotón y gravitón en la página de la Mula Francis de Francis Naukas.
        Estos valores corresponden a un conjunto de experiencias internacionales donde figura el valor límite de la masa del fotón en función de la long de onda. Comprobé que la coincidencia de valores era exacta, no solamente con el valor límite del fotón, sinó que coincidía bien con los valores límite del gravitón.
        Luego me dí cuenta que lo que había tomado como long de onda del fotón era long de onda reducida de Compton y que mis cálculos diferían en un factor 4 pi o 2 pi de los valores experimentales , según como tomara el valor de la long de onda de compton.
        Posteriormente busqué una ecuación que correspondiera a atracción gravitatoria sobre la masa y la hallé en un cálculo que correspondía a obtener el radio de Schwarschild mediante consideraciones Newtonianas en
        http://hyperphysics.phy-astr.gsu.edu/hbasees/Relativ/blahol.html#c2
        Estos resultados diferían de los obtenidos mediante consideraciones relativistas, esa diferencia en el valor se corrigió tomando el valor de masa hallado mediante las ecuaciones mencionadas anteriormente.
        Es decir se obtenía el radio de Schwarzschild como R = 2. G. M / C²

        Actualmente estoy relacionando este valor de Masa con la conservación de momento lineal en el efecto fotoeléctrico y el efecto Compton.
        Ah, también deberia relacionar la masa con la desviación observada en un rayo de luz al pasar en las cercanías de un cuerpo celeste.

      • Héctor Da Silva

        Lamentablemente no veo forma de editar y corregir lo expresado por De Broglie y mal reproducido por mi.
        Saludos

      • Héctor Da Silva

        Jajaja, todo carecerá de sentido si usamos la masa relativista, salvo la conclusión de que el fotón no tiene masa.
        Desde el inicio, sin realizar ninguna deducción elaborada, la fórmula matemática de masa relativista, ajustada por la velocidad:
        m = m0 / raiz cuadrada (1- v²/C²) ,
        donde m0 es masa en reposo,
        cumple: si m0 es 0, m también lo es, y por lo tanto establece conceptualmente que: «Si una partícula no tiene masa en reposo carece de masa.»
        Pero…, no todo es tan evidente, ni siquiera en el lenguaje matemático.
        Existe el punto v=c en la función,donde se produce una indeterminación , y podremos tener un valor distinto de cero, es decir un valor singular de este objeto matemático.
        Naufrago en mis pobres conocimientos al respecto, pero creo, se resuelve hallando el límite de dicha función para v=c.
        En caso de obtenerse un resultado finito en tal punto, podremos decir, tomando en cuenta la función original:
        «Si una partícula carece de masa en reposo no tendra masa a ninguna velocidad distinta de C».
        El fotón podría tener masa a pesar de no tener masa en reposo ,tomando los criterios de la teoría de la relatividad.

        • Manuel Sanchez Carrilero

          En matemáticas cuando aparece una indeterminación del tipo 0/0 puede ocurrir que sea salvable, simplificando la expresión antes de pasar al límite, por lo que tendría límite finito no indeterminado. Por ejemplo la expresión

          (x^2 – 1)/(x – 1)

          cuando x-> 1 es de la forma 0/0, pero se ve fácilmente que antes de pasar al límite indicado, podemos simplificar la expresión, ya que el numerador es diferencia de cuadrados luego tenemos

          (x^2 – 1)/(x – 1) = (x+1)*(x-1)/(x-1) = x+1

          y pasando al límite para x->1 obtenemos 2.

          En otros casos hay que saber como tienden las variables a su límite para saber si son infinitésimos de rango distinto y así salvar la indeterminación.

          Por otro lado la llamada «masa relativista» dada por

          m = mo / SQR(1- v²/c²)

          es una abreviatura como artificio de cálculo que no representa realmente ninguna masa y lo peor es que esa expresión es matemáticamente una forma indeterminada no salvable del tipo 0/0 cuando mo=0 y además sea v=c.

          Por lo tanto que quede BIEN CLARO que dicha expresión no puede ser utilizada para indicar una «variación de masa con la velocidad», únicamente sería un factor que aumenta asintóticamente con la velocidad pero no se traduce en ningún aumento de masa real.

          En el caso concreto del fotón una de las falacias radica en decir que su masa en reposo mo es cero, lo cual es ABSURDO, pues no existe ningún sistema inercial en el que el fotón esté en reposo. El fotón siempre está en movimiento a la velocidad c, y su única energía es siempre cinética, y su momento en función de dicha energía es p = E/c.

          La otra falacia es tratar de utilizar la susodicha expresión para justificar que al fotón le correspondería esa masa y que como resulta indeterminada para mo=0 y v=c, es que existe por ahí alguna forma de salvar el límite, que NO que esa expresión no sirve para el fotón, y para las demás partículas relativistas con v<c es tan solo una abreviatura de cálculo.

          • Héctor Da Silva

            ¿Cómo el fotón puede tener Energía Cinética sin Masa?

            • Manuel Sánchez Carrilero

              Porque no responde a la expresión clásica de energía cinética sino a la concepción absolutamente relativista momento energía.
              Te ruego encarecidamente que te leas mi comentario de fecha 23 enero, 2019 en 0:34, en este mismo cuentos cuánticos, donde se dan cumplidas respuestas a esta y otras dudas.
              Saludos.

            • Debemos interpretar que la energía es siempre lo mismo, el efecto que altera las cosas. ¿Como hace un fotón que se altere el statu de un conjunto de partículas?

              Supón que una partúcula G con gran masa pasa entre muchas partículas cargadas Q, haciendo que algunos fotones de dichas Q se liberen. A su paso se produce un flujo de fotones perpendicular al movimiento de G. Muchos de estos fotones caerán de nuevo en otras partículas Q … un poco más allá en la dirección de movimiento de G.

              Ahora imagina un espacio plagado de partículas G en movimiento y un montón de Q también en movimiento. (La idea del campo Higgs desacoplando carga y masa) En general en un volumen de espacio determinado en un intervalo de tiempo determinado, (cuantificado si quieres) nos encontraremos el mismo número de Qs y Gs al cual llamamos valor neutro.

              ¿Podemos concebir que un conjunto de fotones orbitando una partícula G sea un bosón?

              Es posible que un bosón tenga carga (+) si tiene más fotones en su órbita que el valor neutro, cediendo al medio más que el valor promedio, y otros bosones tendrán carga (-) si tienen menos fotones que el valor neutro cuantificado en promedio.

              En el momento que hay dos cargas opuestas se produciría un campo eléctrico debido al flujo de fotones desde la carga positiva a la negativa hasta que ambas queden en estado neutro. No cabe duda de la compatibilidad de esta idea con lo que conocemos del efecto fotoeléctrico y la relatividad general salvo por un detalle, el flujo de fotones debe «arrastrar» a las partículas G en la misma medida que las G sean capaces de modificar la trayectoria de los fotones. De este modo, tanto las particulas G sin carga (solo con masa) como la carga de las Q (sus fotones) serían portadores de energía.

          • Manuel lo que quieres expresas es correcto. El fotón es el mejor ejemplo de que el modelo clásico aun no es completo. Ni siquiera encaja con Higgs. No obstante, dado que el fotón varía su trayectoria al aproximarse a una masa, es innegable que un fotón puede quedar atrapado en un micro (o nano) agujero negro. Si el fóton no tiene mása tampoco tiene volumen, con lo cual el volumen en el que puede quedar atrapado puede ser infinitesimal, que no quiere decir que sea cero. ¿Es posible extraer un fotón de un agujero negro de esta característica? parece probable, si otra partícula (con mucho más volumen y masa) pasa cerca. La trayectoría del fotón dejará de ser tan cerrada y escapará de la singularidad. Los fotones no interaccionan entre ellos porque no deforman el espacio por lo que en una singularidad, por pequeña que sea, caben cuantos fotones quieras. Parece que no habría razón para pensar que dos singularidades se vean diferentes e interaccionen diferente en función de la cantidad de fotones que atrapen, no obstante, cuantos más contenga más liberará en el mismo lapso de tiempo al aproximarse cualquier otra partícula másica por tanto podríamos decir que esas partículas tienen más o menos carga dependiendo de si están en condiciones de ceder o atrapar más fotones. Ahora podemos imaginarnos todo ese flujo de fotones que van y vienen de unos «agujeros» a otros, y si hacemos un poco de números y simulaciones veremos que en determinados casos se producirán intercambios relativamente estables, y podremos llegar identificar algunas partículas como gluones y piones. Hasta aquí llegamos con el modelo clásico y con mucho derroche de imaginación. No podemos avanzar más si no incorporamos algo que explique el salto de masa y en cierto modo ese es el misterio de que el fotón no tenga masa aun comportándose como si la tuviera al pasar cerca de una partícula con masa.

    • Ese valor de masa del fotón ha sido redescubierto una y otra vez, todas las veces con diferentes argumentos ( puede encontrarse en internet.) La novedad tuya es considerar tal masa el doble.
      Sería conveniente que revisaras los otros aportes con el fin de contrastar tu tesis.

      • No está desencaminado … Por ahí van los tiros. Solo un pequeño apunte … Ni masa ni energía …. la esencia es el Momento.

        • Héctor Da Silva

          Vaya, es la primera vez que hay un mínimo comentario de aliento, ya estaba creyendo que lo que decía era una tremenda estupidez que no merecía ninguna consideración, más teniendo en cuenta que mis conocimientos de física no son gran cosa, ni que hablar de física cuántica.
          Vea en la página de wikipedia sobre la long de onda de Compton, el cuadro sobre las condiciones que cumplen las partículas másicas.

        • Héctor Da Silva

          Podemos establecer la siguiente relación, E=1/2mv²+m0 C², válida aún a vemocidad de la luz, para m = masa no inercial o masa newtoniana, (paciencia que ya la justificaré) y Energía cinética = p²/2.Masa no inercial, para la física clásica y fotón másico.
          No tengo noción del alcance de esta relación.

      • Héctor Da Silva

        Conozco perfectamente los otros valores de masa obtenidos como M = h / long onda . C, mientras el obtenido por mi es M = 2 h / long de onda . C, pero en principio este último correspondería al valor correcto de masa gravitatoria del radio de Schwarzschild obtenido mediante consideraciones relativistas, pero en mi caso lo obtengo para la luz como calcularía una orbita de un cuerpo másico mediante ecuaciones de Newton a velocidad constante C.
        En la página que menciono sobre agujeros negros se calcula con la masa generalmente dada pero se quedan cortos en un factor 2, justamente mi masa es el doble de la mencionada.
        No chequeo el valor de la masa que calculo contra los otros valores mencionadas, sinó que busco resultados teóricos y experimentales que me establezcan si el valor que he calculado es correcto o no.

  9. Hola: ¿como es que una piedra de fluorita que esta en reposo tiene luz? esa luz no tiene masa? también ocurre con fuorita sintetica. o con la pintura fluorescente etc.la luz solar no se mueve en el vacio sino en la materia del aire. si no tuviera masa como podría impulsarse?

    • Hummm… una piedra de fluorita no «tiene» luz. Emite luz que no es lo mismo. Y recuerda que la luz del sol nos llega desde el sol atravesando 150 millones de kilómetros de vacío.
      Lo que ocurre es que la luz no es materia. Es una fluctuación de un campo.

  10. Manuel Sanchez Carrilero

    Por si es de utilidad para alguien transcribo un nuevo comentario sobre el tema:

    Una primera misión es encontrar una expresión para la energía cinética en el campo de la relatividad dada por

    (1) Ec = ƒFdr = ƒ[d(mv)/dt]dr = ƒv d(mv)

    donde la integral ƒ está extendida entre 0 y v

    Una integración por partes, llamando “masa relativista”, como después justificaremos, a la expresión

    (2) m = mo/SQR(1-(v/c)^2)

    nos conduce, después de algunas simplificaciones sencillas, a la relación

    (3) Ec = mc^2 – moc^2

    donde mo es la masa en reposo, es decir el valor de la masa cuando v=0.

    La anterior la reescribimos en la forma

    (4) mc^2 = moc^2 + Ec = E

    donde E es la energía total, suma de la energía en reposo y la energía cinética. El haber concentrado la relación con la masa en reposo y la velocidad en la “masa relativista” nos permite poner la energía total en función de este parámetro por la expresión

    (5) E = mc^2

    Como hasta ahora hemos supuesto que v0 sea v→c entonces el valor m tiende a infinito así como la energía E y el momento p, es decir se convierten dichos valores en asintóticos, conforme crece la velocidad.

    Pero es más, las cosas no acaban aquí: cuando, como en el caso del fotón, sea mo=0 esto implica que v=c, con lo que todas las variables comentadas en el párrafo anterior m, E, p se convierten en indeterminaciones del tipo 0/0.

    Entonces habrá que eludir esta situación haciendo lo siguiente:

    Elevar al cuadrado la (5) y multiplicando por c la (6) y elevar también al cuadrado, y restar en ese orden los resultados, se llega sin ninguna dificultad a la expresión

    (7) E^2 = (mo^2)*(c^4) + (p^2)*(c^2)

    Hemos obtenido así una expresión que contiene solamente la masa invariante mo, llamada en reposo, para cualquier partícula relativista con mo distinta de cero y v<c, más la energía y el momento.

    En el caso del fotón llamar a mo masa en reposo es absurdo ya que no existe ningún sistema inercial en el que el fotón se encuentre en reposo. Por eso es mejor llamar a mo una constante en cada caso que, sin rodeos, sería la masa de la partícula.

    Así pues la anterior expresión (7) es válida también para el fotón, poniendo que su masa mo es cero y extrayendo la raiz cuadrada obtenemos

    (8) E = pc

    por lo que el momento del fotón vendría dado en función de su energía por la

    (9) p = E/c

    Vemos que a pesar de ser cero la masa del fotón, la (4) nos indica que toda su energía es cinética con un momento dado por la (9).

    Nos damos cuenta también que en (5) el cociente E/(c^2) tiene dimensiones de una masa y lo llamamos por brevedad “m”, haciendo notar que lo podríamos haber llamado con cualquier letra, para evitar cualquier connotación con una masa real. Así pues la (6) se pondría escribir como

    p=(E/c^2)*v

    al poner v=c para el fotón, obtenemos lógicamente la ya mencionada (9).

    Aquí aparece una connotación histórica, para los defensores de la “antigua usanza” esa abreviatura era precisamente la llamada “masa relativista”, cosa prohibida por los revisionistas y fanáticos de las partículas que solo defienden la existencia de la masa mo (masa verdadera y la única a considerar) y como solo es esta ya no sería necesario el subíndice, pasando a llamarse con m a la masa.

    De aquí que expresiones como las (2), (5) o (6), ellos añaden, introdujeron en la historia de la Física más confusión que claridad, basándose aquellas en que servían para mantener la analogía con las fórmulas clásicas.

    Es más y que sirva solo como ejemplo, la famosa ecuación que Einstein escribió, con la notación de este comentario, sería

    Eo = mo c^2

    es decir, se trata de la energía en reposo que todo cuerpo de masa mo constante posee, que se añadirá a la energía cinética que corresponda en virtud de su velocidad.

    Cuando se hablaba de los aceleradores de partículas se decía que la (2) implicaba un aumento relativista de masa por las grandes velocidades puestas en juego; hoy habría que decir que no existe tal aumento de masa, sino que las partículas sufren un gran aumento de energía dado por el factor E/(c^2).

  11. Disculpe, hay algo que no entiendo, y es que si los fotones no tienen masa pero si energía y las unidades de la energía son el Joule (o electrón-voltio) la cual tiene como unidades en el Sistema Internacional (SI) «Kg*m^2/s^2».
    Entonces cómo es posible que las unidades en el SI de la energía se encuentre las unidades de la masa, si los fotones no tienen masa.

  12. Ing Juan Jose Gonzalez Rodriguez

    Como decia Ampere de la simple deduccion y observacion de los hechos he llegado a la conclusion q las lineas de fuerza son fotones compartidos por las cargas electricas o magneticas
    Si requeris mas informacion estoy a tus ordenes

  13. No estoy de acuerdo.
    Si el futón no tiene masa, no puede quedar atrapado en el campo gravitatorio de un agujero negro y por tanto la luz pude escapar a su atracción. Tampoco puede ser desviada de su trayectoria por la acción de la gravedad por tanto no pueden existir los remolinos de luz…

    … O el fotón tiene masa.

    De pequeño nos demostraban matemáticamente que 2+2=5. El truco consistía en introducir una fracción 0/0 que transformaban en 1 con el argumento de que en una división cuandol dividendo y divisor son iguales, el cociente es siempre 1.
    No cuela… y hay mucho más.

  14. Nada disculpa, que lo he vuelto a leer y veo que en esa segunda explicación se confirma que E=mc2 no se puede aplicar al fotón…

    • Manuel Sánchez Carrilero

      Para Anónimo 20mar2018

      La expresión E=mc^2 es válida en todos los casos incluido el fotón.

      Si realmente quieres aprender de forma asequible y aclarar tus dudas, te ruego consultes, en este mismo espacio, mis respuestas, entre otras, de fechas 14ago2017 y 13feb2017.

      Allí viene todo lo que preguntas; no obstante si necesitas alguna aclaración más no dudes en preguntar.

      Saludos

  15. Hola, en la prima parte explicas como a llega a las ecuaciones de la energía y se llega a la conclusión que para partículas que se mueves a v=c E=pc.. y que para partículas a velocidades menores que c, E2=m2c4-p2c2 y a partir de esta se indica que si la partícula está en reposo E=mc2 y que por tanto esta ecuación solo es aplicable a partículas en reposo. Esta parte me quedo clara, pero más abajo dices «en el caso extremo del fotón» E=mc2 es energía cinética… pero no hemos quedado que esa expresión solo sirve para partículas en reposo y el fotón no está en reposo??? esto me ha descolocado.

  16. Hola, buenas. Tengo una duda.
    La masa del foton viene dada por su momento lineal.Hasta aquí bien. Mi duda viene ya que el momento lineal es el producto de la masa y la velocidad y si el foton consigue su masa a partir de su momento y no tiene masa en un principio, el producto del momento da 0…… Esto es lo que me confunde. No se si me he explicado bien. Espero respuestas para aclarar. Gracias

    • Manuel Sánchez Carrilero

      Por favor Juan Manuel, lee detenidamente el resumen que he redactado para ti tratando de aclarar tus dudas. El resto creo lo deberías completar leyendo otros comentarios de este interesante blog.
      ¡Ahí va!

      Por medio de un proceso de integración adecuado se llega a demostrar que

      E = Eo + T (1)

      donde E es la energía total relativista, dada por la expresión, igualmente demostrable

      E = m c^2 (2)

      La energía en “reposo” la da el sumando Eo = mo c^2, donde la constante característica en cada caso es el valor mo llamado a veces masa en “reposo”. Finalmente T es la energía cinética relativista.

      En el proceso de demostración anterior, interviene la expresión del aumento de masa relativista con la velocidad, dada por

      m = mo/sqr(1-(v/c)^2) (3)

      donde la masa en reposo mo aumenta al valor m cuando la velocidad lo hace desde cero hasta v.

      El paralelismo de expresiones relativistas con las habituales de la mecánica clásica, puede conducir a equívocos. Por ejemplo el momento lineal, prescindiendo del aspecto vectorial para simplificar, venía dado por

      p = mv (4)

      siendo m la masa de la partícula con v su velocidad.

      Para una partícula relativista, análogamente valdría la misma expresión, pero la masa se corresponde con la expresión relativista anterior (3).

      Para el caso extremo del fotón, cuya velocidad es v = c, y cuya masa en “reposo” constante es mo = 0, se cumplirá: primero, en (1) toda la energía E=m c^2 es cinética, ya que la energía en “reposo” es nula; segundo, por la (3) su masa m dará una indeterminación del tipo 0/0, toda vez que mo=0 y v=c. Por lo tanto, dicha fórmula no puede ser aplicable al caso del fotón. Es decir no existe ningún sistema inercial en el que el fotón esté en reposo. De aquí que al intentar aplicar la (4) no llegaríamos a un resultado correcto para el valor de p.

      Por el contrario, la expresión (2), E = m c^2, que da la energía total, nos permite escribir, teniendo presenta la mencionada (4), en la que hemos supuesto v = c, lo siguiente:

      E = m c^2 = mc c = p c

      de donde llegamos a la

      p = E / c (5)

      Por lo tanto, el cálculo del momento del fotón exige conocer su energía y de aquí, por la (4), será

      m = p/c

      sin perder de vista que dicha masa obtenida del momento p lo es a través de la (5); en definitiva sería

      m = E/(c^2)

      Existen muchos fenómenos en los que su naturaleza cuántica permite igualar la energía total de Einstein con la de Planck, es decir

      m c^2 = hf

      lo que nos daría la ”masa” de los fotones en función de la frecuencia f de la radiación emitida y de las constantes h de Planck y velocidad c de la luz, por la relación

      m = hf/(c^2)

      Notas de nomenclatura:
      x^2 significa x elevado al cuadrado
      sqr ( x ) significa raíz cuadrada de x
      Eo y mo la o añadida significa subíndice

  17. Manuel Sánchez Carrilero

    La energía relativista viene dada por la expresión:
    m c^2 = mo c^2 + T (1)
    siendo E = m c^2 la energía total relativista, donde m es la masa de la partícula.
    Dicha energía se descompone en un sumando denominado energía en reposo, en función de la llamada masa en reposo mo, que es una constante característica de cada partícula y sería el valor de m cuando v=0, es decir cuando dicha partícula está en reposo respecto al observador.
    El segundo sumando T es la energía cinética relativista.
    Notar aquí la primera falacia en la que incurren algunos usuarios cuando dicen que la expresión E=mc^2 solamente es una fórmula de equivalencia entre una energía y una masa, sin reparar en que la expresión (1) es obtenida por un proceso de integración fuerza desplazamiento, teniendo en cuenta la (2).
    Todo esto es consistente con la expresión relativista de la masa en función de las velocidades dada por
    m=mo/SQR(1-(v/c)^2) (2)
    que nos dice que la masa m en movimiento a la velocidad v es mayor que la masa en reposo mo.
    ¿Qué sucede si se trata del fotón? Como la masa en reposo para el fotón es mo=0, la primera ecuación (1) dará
    mc^2 = T
    es decir la energía total es toda cinética, y dicho fotón puede moverse solo con la velocidad de la luz y nunca puede estar en reposo en un sistema inercial.
    La segunda ecuación (2), poniendo mo=0 y también v=c, dará una indeterminación del tipo 0/0.
    Antes de seguir relato la segunda falacia: La velocidad de la luz c es siempre constante cualquiera que sea el observador. Debe ser reemplazada por: La velocidad de la luz c es siempre la misma para todos los observadores inerciales.
    Veamos la vidriosa cuestión de que la masa del fotón es siempre cero: me remito a la página 460, del libro La Gran Ilusión, en el capítulo: dentro y fuera del ascensor, donde la voz autorizada de Stephen Hawking, dice “Un haz de luz posee energía y la energía tiene masa…” Se está refiriendo al famoso experimento mental del ascensor de Einstein, en que la luz adquiere curvatura tanto en un campo gravitatorio, como en un ascensor acelerado, todo basado en la equivalencia exacta entre masa gravitatoria y masa inerte (en este caso del fotón).
    Otro relato, página 514, Física Teórica, vol. 2, donde J. Cabrera y Felipe dice “…tengamos en cuenta que la masa correspondiente a un fotón es hf/(c^2)…” Se está refiriendo al choque de un fotón de frecuencia f con otra partícula (electrón), como sucede en el efecto Compton.
    Conclusiones: la luz, el fotón, tiene masa NO NULA que se obtiene al despejar m de la igualdad de energías, relativista y de Planck, según la relación mc^2 = hf.
    Por otro lado, vuelvo a reafirmarme en que lo que sí es cero es la masa mo del fotón, es decir la masa en reposo. Es más si tratáramos de calcular la masa por la relación relativista en función de las velocidades vista arriba (2) obtendríamos 0/0 como ya dijimos, porque esa fórmula relativista es válida para cualquier partícula con v<c.

  18. Hola, soy nuevo por acá y me gustaría hacer una pregunta con ralación al tema. Siendo el fotón carente de masa, _Se prodría decir entonces que es un fenómeno el cual no tiene las características de los cuerpos de 4 dimensiones (ancho, largo, profundidad, tiempo)…Acá la pregunta> Cuántas dimensiones tiene un fotón? Post data: trabajo en la teoría de que quizás el mismo se origine en dimensiones inferiores a la nuestra, quizás una de dos; una de ellas puede ser el tiempo y otra que pueda darle existencia no espacial, o mas bien referencial como lo sería un punto. Qué dicen ustedes? Gracias.

    • Santiago Avilés Quevedo

      Tenemos que recordar: Se llama materia a lo que existe y constituye de modo real y objetivo los cuerpos que generan el espacio inconmensurable.
      La materia existe en la forma de átomos aún no bien conocidos, los cuales son las partículas primigenias portadoras de todas de todas las propiedades fundamentales de la materia. Esta proposición es una hipótesis de trabajo que deberá recibir confirmación experimental posterior.
      La cuestión de las propiedades de la materia se aborda con el análisis de sus manifestaciones para encontrar las que tiene carácter general, en el sentido de que están presentes en toda la materia.
      Lo que se observa es que todo está en movimiento (dinámica) y que en la mayor parte de la materia que se percibe, está la tendencia al reposo (estática). Esto es el punto de partida de este análisis.
      Esta observación sugiere la existencia de materia con una de las dos propiedades mencionadas. La dinámica o la estática. Esta es una cuestión crítica ya que determina un cambio drástico en el concepto de lo que existe.
      Según esta noción, la materia es no solo la masa, lo que pesa, si también los es, lo que genera el movimiento y es dinámico en todas sus manifestaciones.
      Se propone, entonces, que esto que genera el movimiento, es algo a al o que legítimamente se puede llamar materia, una de cuyas propiedades es la energía, que ahora se mide con el trabajo realizado. Un vez que actúa. A esa materia con energía se le llamará MATERIA DINAMICA, en lo sucesivo. Se le concibe como lo que genera las fuerzas dinámicas y vitales de la naturaleza y como responsable o causa de la totalidad de los fenómenos energéticos y vitales, y de la constitución y comportamiento de los seres vivos dinámicos y los no vivos y del fenómeno evolutivo.

      Vázquez-Reyna, Mario (1998). Reflexiones en torno la materia, la energía y la masa. Anexo B. Cd. De México. ISBN 970-91797-1-3

      El Fotón es materia dinámica.

      • Mucho rollo para no decir nada. Pasemos totalmente de la tabla periódica de los elementos y centrémonos en las partículas subatómicas. ¿De qué están hechas? Ya sabemos que no todas son elementales ¿Hay otra tabla de elementos dentro del átomo?
        Cómo puedes ver, resolver una pequeña cuestión, conduce a otras mucho más grandes.
        El fotón tiene masa. La pregunta es ¿Que es masa? Y la respuesta que buscamos no es la canónica sino de qué se compone la masa, de que está hecha. No vale decir de harina y agua (y levadura) :p

  19. Pingback: El Bosón de Higgs – Astro Gredos

  20. Santiago Avilés Quevedo

    Materia.— Elemento increado, universal, que da constitución real y objetiva a todos los seres inorgánicos y orgánicos, sea ó no que ellos hieran nuestros débiles sentidos, ya que nuestros deficientes instrumentos de observación científica puedan ponderarlos, ya que sólo tengamos concepto de ellos por los datos de la inducción lógica, cual sucede con el imponderable eter. El universal elemento está dividido en dos polos: luminoso-dinámico (energía ) y sombrío-estático (masa ). De estas propiedades fundamentales y mecánicas derivan todas las que en múltiples, varías y complexas combinaciones manifiestan todos los seres que actúan en el Cosmos.
    Jesús Ceballos Dosamantes, 1897.
    https://archive.org/details/cienciayreligio00dosagoog

  21. Santiago Avilés Quevedo

    Se llama materia a lo que existe y constituye de modo real y objetivo los cuerpos que generan el espacio inconmensurable.
    Lo que se observa es que todo está en movimiento (dinámica) y que en la mayor parte de la materia que se percibe, está la tendencia al reposo (estática). Esto es el punto de partida de este análisis.
    Esta observación sugiere la existencia de materia con una de las dos propiedades mencionadas. La dinámica o la estática. Esta es una cuestión crítica ya que determina un cambio drástico en el concepto de lo que existe.
    Según esta noción, la materia es no solo la masa, lo que pesa, si también los es, lo que genera el movimiento y es dinámico en todas sus manifestaciones.
    Se propone, entonces, que esto que genera el movimiento, es algo a al o que legítimamente se puede llamar materia, una de cuyas propiedades es la energía, que ahora se mide con el trabajo realizado. Un vez que actúa. A esa materia con energía se le llamará MATERIA DINAMICA, en lo sucesivo. Se le concibe como lo que genera las fuerzas dinámicas y vitales de la naturaleza y como responsable o causa de la totalidad de los fenómenos energéticos y vitales, y de la constitución y comportamiento de los seres vivos dinámicos y los no vivos y del fenómeno evolutivo.
    De la misma manera, se reconoce a la masa con la materia con inercia, por lo cual se opone al cambio de movimiento y al movimiento mismo. A esta materia con inercia se le llamará MATERIA ESTÁTICA, en lo sucesivo. Se le concibe como lo que genera las fuerzas estáticas y opuestas a la vida en la naturaleza y como responsable o causa de la totalidad de los fenómenos inertes y contrarios a la vida, y de la construcción y comportamiento de los seres antagónicos a la vida, y al fenómeno evolutivo. También posee la propiedad de ser extensa.
    La materia dinámica y la materia estática no se crean ni se destruyen.
    La radiación electromagnética se considera ser materia preponderantemente dinámica, es decir, puede una porción estítica y la radiación constituida de partículas con masa, contiene la componente estática que proviene de la masa.
    La propiedad de generación de reposo es exclusiva de la materia estática. Esta genera el reposo, poniéndose en reposo ella misma e impidiendo el movimiento de aquello que la rodea y que puede retener.
    Estas dos últimas propiedades son contradictorias entre si y constituyen la raíz causal de la contradicción universal. No coexisten en el mismo átomo.
    Cada átomo posee como atributo o propiedad una u otra de estas dos propiedades.
    Esta formulación de las propiedades de la materia se desprende de los principios de conservación, en cuanto que siendo increada la materia, ha sido siempre como lo es actualmente. Es decir, que sus propiedades fundamentales deben tener la calidad de inmutables, o como se menciona en el lenguaje de la Física, son invariantes frente al tiempo y respecto de transformaciones de todo tipo, en caso de que se les describiera con respecto a un marco de referencia. En cambio, en su devenir temporal espacial, su manifestación cambia según el entorno, la circunstancia concreta en que se encuentran los átomos, y el nivel de integración evolutiva que haya alcanzado.
    Así la materia tiene dos propiedades fundamentales, la propiedad espacial y la propiedad de general movimiento, la materia dinámica o la propiedad de generar reposo la materia estática. El componente mínimo de la materia, el átomo, no el átomo del elemento químico, resulta ser una entidad impenetrable a muy cortas distancias y penetrable a grandes distancias, es capaz de ejercer fuerzas hacia fuera de sí mismo, y de asociarse con otros átomos bajo ciertas condiciones. Las propiedades dinámica y estática de generar movimiento o reposo respectivamente, son la raíz causal de la esencia o naturaleza de los correspondientes átomos o de la correspondiente materia.
    Vázquez-Reyna, Mario (1998). Reflexiones en torno la materia, la energía y la masa. Anexo B. Cd. De México. ISBN 970-91797-1-3

  22. Va mucho tiempo de este artículo, ojalá alguien lea este comentario. Primero agradecer por la entrada y por todo el blog en general: me encanta y se lo recomiendo a toda las personas que conozco y que quieren saber más de física.

    Acá va mi duda: ¿Sería apropiado pensar que la energía se divide en una parte asociada al movimiento, el momentum, y otra asociada al reposo, la masa? Creo que sí, pero eso no es tanto lo que me interesa. Mi interés está en la inercia. La inercia solo está dada por la masa (por ese componente de la energía que no está asociado al movimiento y que me parece razonable pensar que no es otra cosa más que energía potencial) y la inercia es la resistencia a cambiar de velocidad, o sea la resistencia a cambiar la cantidad de la otra parte de la energía. Podría pensar que el espacio-tiempo tiene una resistencia a cambiar su forma y que esa resistencia devendría en la inercia, pero no solo la masa curva el espacio-tiempo, sino toda la energía, lo que contradice mi hipótesis que es la resistencia del espacio-tiempo a cambiar su forma lo que se traduce en la inercia, puesto que de ser así los fotones, que también curvan el espacio-tiempo, también tendrían inercia.
    Entonces –y ahora sí va mi duda–, ¿tenemos alguna explicación convincente de por qué solo la masa tiene inercia?
    Entiendo que se podría considerar que decir «el espacio-tiempo se resiste a cambiar su forma y eso da la inercia» no es muy distinto a decir «la masa es una propiedad de los cuerpos con masa», pero me parece que se puede seguir indagando bastante en el porqué de la segunda afirmación, mientras que la primera (o alguna similar) es más fundamental y por lo tanto, a mi juicio, más factible de asumir como una propiedad sin preguntarse el porqué.

    Saludos y gracias nuevamente por dar vida a este blog.

  23. Entonces, si no tiene masa no es materia, solo un tipo de energía sin materia, por tanto estaría mal clasificarlo como una partícula elemental o como particula física o química en sí.
    Tal como existen las 4 fuerzas Nuclear fuerte, debil, electromagnetismo y gravedad; se les llama a cada unidad de energía sin masa como gluones, bosones, fotones y gravitones. Pero aun juntando estas cuatro fuerzas no puedes crear materia, puesto que solo son energía sin masa. Entonces NO es correcto llamarlas particulas elementales.
    Caerían en un termino nuevo que debería ser separado de la materia y concebido como partículas de energía pura, ya que tampoco se pueden contener sin ser transformadas por materia.

    • En el cuarto eje espacial, j.c.t, podriamos obtener una velocidad c, com masa m, la cual no variaria nunca porque siempre se va a la misma velocidad c. Variaria la masa de los otros tres ejes, pero no la del cuarto.

  24. La luz esta formada por fotones, esta es desviada por la gravedad, la gravedad solo interactua con cuerpos con MASA por lo que los fotones han de tener masa

    • Un fotón en reposo no sería atraído por un agujero negro. Sí se mueve a la velocidad de la luz la cosa cambia.

      • Vale pero que me dices de la formula E=mc^2 segun Einstein. Si el foton no tubiese masa no tendria energia cuando si que tiene😋✌

        • Esa fórmula no es la completa, ademas cualquier masa en reposo debe ser diferente a cero para que la fórmula sea aplicable, a no ser que consideremos la posibilidad de masa negativa, cosas más raras se han visto.

          En cualquier caso la madre del cordero esta en responder a eso ¿porque algo que en reposo no tiene masa pasa a tener masa si se mueve a la velocidad de la luz?¿O quiza no sea así exactamente?

    • Lo que realmente pasa es que la gravedad curva el espacio-tiempo, y al estar la luz dentro del espacio-tiempo; es obvio que ésta se curvará por la gravedad.

    • Einstein lo dejó bien claro. Los Fotónes no tienen masa, por tanto la gravedad no les afecta. La gravedad solo afecta a los cuerpos que la tienen y este efecto se manifiesta como una deformación espacio-temporal. Los fotónes solo siguen la trayectoria del espacio tiempo por el cual se propagan. De ser así como propones, la teoría cuántica gravitatoria estaría resuelta porque sabríamos cómo se relaciona la gravedad con los fotones al igual que lo hace el efecto fotoeléctrico. A decir verdad los fotones ni si quiera tienen carga o una energía intrinseca, esta depende de ciertos atributos que son consecuencia de otros factores. Para que te hagas una idea de qué es realmente un fotón como una radiación de onda-partícula(depende el caso) o boson mensajero que (no genera, si no que mas bien) trasfiere energía. Es todo. Te pego un comentario que hice hace tiempo en otro lado donde trato de explicarme mejor al respecto y el por qué un fotón no tiene masa respecto a la Teoría de la Relatividad de Einstein.

      No soy físico aún ni nada, pero estudio el tema desde algún tiempo. Sencillamente en lenguaje llano te puedo decir que NO TIENEN MASA. Me explico; Puesto que la luz al estar formada por fotones viaja a la velocidad máxima solo alcanzable por energía pura al nivel mas elemental. Nada con una masa en este universo o por lo menos con tres dimensiones puede viajar más rápido que un fotón de acuerdo con la Relatividad de Einstein expresada en la fómula E=MC2. (esta nos dice que la energía pura en este caso el fotón, sería el producto de una masa acelerada a la velocidad de la luz o viceversa. «Una puede ser convertidad en la en la otra, de hecho una mepequeña cantidad de masa puede contener una gran cantidad de energía (que se disipan en forma de ondas que viajan a la velocidad de la luz…»fotones»…)(pasa todo el tiempo dentro de los colisionadores de partículas y en las explosiones nucleares)…O en caso contrario al acelerar una masa, y para que esta pueda llegar a la velocidad de la luz, requeriría de una energa infinita que como sabes es algo imposible. Puedes ver en la misma ecuación que cada vez que la masa se acerca a la velocidad de la luz la energía necesaria para acelerar se multiplica exponencialmente por dos. (M=E/C2). Es decir, nunca llegarías a tener toda la energía necesaria porque mientras mas cercano a la velocidad de la luz estés, más energía necesitarás… Por tanto los fotones NO TIENEN MASA y por eso alcanzan la velocidad de la luz o la fotónica, que es lo mismo… De hecho las ondas electromagnéticas no tienen masa, la gravedad no les afecta y es un fenómeno que viaja a la velocidad de la luz, su partícula mensajera es el fotón. Simplemente imagínalos como objetos de dos dimensiones (existen como entidad en el tiempo y tienen existencia en otra dimensión adicional).

      • Si fueras un fotón creerías que estas quieto y que todo lo demás se mueve. Verías como algunas cosas (muy pesadas) varían su trayectoria para aproximarse a ti y concluirías que tienes mucha masa.

        Observarías también que cuando dos grandes masas equidistantes se aproximan a ti (de tal modo que pasará una a tu derecha y otra a tu izquierda) no se produce el mismo acercamiento y concluirás que tienen mucha menos masa que otras, o que hay alguna carga que hace que se repelan, o que …. ¡eres tú el que se mueve!

        Lo que quiero decir con esto es que todo lo que observamos es relativo a nuestra perspectiva. Según la perspectiva cuántica el fotón no tiene masa cuantificable; quizá haya otro modelo en el que cuadre que los fotones tengan masa, o quizá el concepto de masa no tenga sentido en ese otro modelo.

  25. Pingback: Si sos inteligente y no lo sabes deberias saberlo - Chismes Mundo | Chismes Mundo

  26. Pingback: Cuantica y relatividad…..¿Porqué no? | Dorian Main Coffe

  27. Víctor Alejandro

    Yo solo quiero saber que alguien me de una respuesta contundente de esta pregunta porfavor.

    ¿Porqué el foton se mueve en forma ondulatoria y no lineal?

    • se mueve en forma ondulatoria porque un foton es un campo eléctrico que genera un campo magnético y este vuelve a generar un campo eléctrico.

    • Los fotones se mueven en linea recta, así que se mueven en forma lineal. Si lo que preguntas es porque son una onda o se mueven como una onda es porque realmente son una onda.
      Los fotones son un campo eléctrico, que en movimiento genera un campo magnético y este campo magnético en movimiento genera un campo eléctrico., y así sucesivamente.
      y entonces¿por qué es una partícula? porque la energía no puede tomar cualquier valor, sino que esta cuantizada.

  28. Perdonen, tengo una duda; si una partícula tiene energía, pero no tiene masa, la ecuación E=M c2 podría reducirse a E= c2. ¿Esto significaría que toda partícula sin masa, pero con energía, saldría automáticamente disparada a la velocidad de la luz?
    Por otra parte, si junto a esa partícula, ponemos otra con el doble de energía… dado que no puede ir al doble de la velocidad de la luz, ¿no significaría esto que toda particula sin masa que se moviera a la velociadad de la luz, adquiriría la masa correspondiente a su energía?

    • Si la partícula no tiene masa m=0 y por lo tanto E=0 siempre que esté en reposo. La fórmula completa, como se ha dicho por ahí es

      E²=m²c⁴+p²c², así si una partícula no tiene masa m=0, puede tener energía debida al movimiento. Recuerda que el movimiento está representado por la cantidad p que es el momento lineal.

      Las partículas sin masa están condenadas a moverse siempre a la velocidad de la luz por lo tanto si tienes el doble de energía significa que tienes el doble de momento, nada más, nada menos.

    • Mira estuve leyendo algunas respuestas que te dieron y me parece que hay ciertas interpretaciones quizás erróneas (con todo respeto). Quisiera opinar al respecto lo siguiente sobre tu pregunta de una forma llana y sin ver muchas fórmulas que no comprendas; …Los dos lados de la igualdad en fórmula de Einstein significan lo mismo. Energía=Masa acelerada al cuadrado la velocidad de la luz. Son dos aspectos de una misma cosa. Pero no puedes separar la energía de la masa así como lo hiciste, lo ideal sería que la masa pasara a dividirse entre la energí, porque del otro lado de la igualdad estaba multiplicandose, así; C^2=E/M. Eso diría que para alcanzar una velocidad mas alta que la de la luz, tendrías que el resultado de dividir la energía total de tu cuerpo y la masa que contiene tu cuerpo. Puesto que la ecuación estaba igulada de un lado y de otro en la ecuación original, osea había una igualdad los resultados de la energ[ia con respecto a la masa era el mismo, por tanto el resultado serían sin sentido, algo como una relación 1/1=1. EN POCAS PALABRAS LA ENERGIA QUE NECESITAS PARA ALCANZAR UNA VELOCIDAD (en este caso el cuadrado) MAS ALTA QUE LA DE LA LUZ ES INFINITA. Por otro lado, ninguna partícula rompe su estado y empieza a perder su masa original para irse transformando en energía si no la aceleras con algo de energía. Pero si lo haces bien pienso que es cierto y lógico tu razonamiento: una partícula con masa cero, como los son el fotón, el gluón o el todavía no confirmado gravitrón, son pura energía por tanto alcanza la velocidad máxima en nuestro universo de 4 dimensiones. (Pero no es que ganen energía y llegen hasta allí, no, su velocidad siempre es constante) De hecho, en las ya confirmadas ondas gravitacinales, la radiación electromagnética y la luz, viajan en el vacio exactamente a la misma velocidad; la velocidad de la luz…NINGUNA TIENEN MASA, solo son partículas mensajeras de la fuerza. Respecto a la segunda pregunta: -Toda particula sin masa que se moviera a la velociadad de la luz, adquiriría la masa correspondiente a su energía? La respuesta es que volviendo a la fómula, TENDRíAS TODA LA RAZóN POR ASUNTO DE LA MISMA EQUIVALENCIA, pero sucede que en las partículas mensajeras antes mencionadas, la velocidad es constante no es que adquieren energía y mientras mas velocidad o viceversa. Pensando lo inverso osea, el decaimiento de la velocidad de ellas (pérdida de energía) se transformaría de forma automática en masa. NO FUNCIONA DE ESA MANERA. Como sabemos hoy en día la masa se debe a la interacción de algunas partículas con del campo de Higgs y cuya partícula mensajera es el Bosón de Higgs justamente. Esto lo tomé de Wikipedia: …Esta teoría sugiere que un campo impregna todo el espacio, y que las partículas elementales que interactúan con él adquieren masa, mientras que las que no interactúan con él, no la tienen.(QUIZAS POR LAS ALTAS VELOCIDADES A LAS QUE SE MUEVEN, NO LES PERMITEN ADQUIRIR MASA). En particular, dicho mecanismo justifica la enorme masa de los bosones vectoriales W y Z, como también LA AUSENCIA DE MASA DE LOS FOTONES. Tanto las partículas W y Z, como el fotón son bosones SIN MASA PROPIA, los primeros muestran una enorme masa porque interactúan fuertemente con el campo de Higgs, y el fotón no muestra ninguna masa porque no interactúa en absoluto con el campo de Higgs, Entonces, las partículas u ondas NO POR SU DECAIMIENTO DE ENERGIA CINETICA son CONVERTIDA AUTOMáTICAMENTE EN MASA. Espero haber aclarado tu duda.

  29. Manuel Sánchez Carrilero

    La expresión relativista para la energía es E=mc^2, donde m es la masa del fotón. La expresión de esa misma energía teniendo en cuenta la mecánica cuántica vale E=hf, donde h es la constante de Planck y f la frecuencia. La igualdad de ambas expresiones conduce a que la masa del fotón vale m=hf/c^2; es decir la masa depende linealmente de la frecuencia. Digamos que los fotones “violetas” tienen mayor masa que los “rojos”. En el vacío todas las frecuencias de un mismo rayo de luz viajan al unísono a la velocidad c, la luz es blanca que es lo que distingue el ojo del observador. Cuando ese mismo rayo penetra en un medio transparente, vidrio por ejemplo, cada ”color” viaja a una velocidad distinta, por supuesto inferior al valor c, y con dirección ligeramente también distinta. Se produce el fenómeno de la dispersión de la luz explicado por Newton y de todo el mundo conocido.
    Lo explicado más arriba tiene sentido para que la expresión que da la masa de una partícula en función de su velocidad: m=mo / Raiz cuadrada ( 1 – v^2/c^2 ), donde mo es la masa para una velocidad v=0, no conduzca a un absurdo. En efecto, si tratamos de calcular la masa de la partícula para una velocidad v=c, obtendríamos una indeterminación de la forma 0/0. Es decir la fórmula no es válida para calcular la masa en este caso, debiéndose utilizar la del párrafo primero de arriba.
    Pero la masa en “reposo” mo tiene una gran utilidad. Por ejemplo, utilizando la igualdad del párrafo primero y el valor de mo, llegaríamos a la conclusión de que la distancia al cuadrado de la interacción es inversa de dicha masa. Por eso para el fotón (mo = 0) daría una distancia infinita: en efecto el alcance de los efectos electromagnéticos se extienden hasta el infinito. Por el contrario, en la interacción fuerte la distancia es muy pequeña porque las partículas implicadas tienen una masa en reposo mo muy grande.

    • Sólo una observación: las imágenes que vemos no existen fuera de nuestro cerebro, ya que el ojo humano no distingue luces ni colores, sólo radiaciones en distintas frecuencias. Es nuestro cerebro el que transforma las radiaciones que captan nuestros ojos en imágenes dotadas de luz y color.

  30. «Todo existe y no existe»

  31. Interesante discusión, como soy lego en el tema, de antemano pido disculpas, pero a veces una observación simple puede traernos algo de claridad donde a veces las tinieblas abundan; Mi pregunta es: ¿Cuál es la razón por la cual los fotones invariablemente se alejan de su fuente de emisión?, O, al revés, ¿Por qué ellos no se acumulan al lado de afuera de su fuente de emisión?. si carecen de masa ¿De donde obtienen su asombrosa velocidad?, ¿Son ellos antigravitacionales?.

    Agradecido de vuestras respuestas.

    • El espacio-tiempo; es decir, aquello que se deforma y hace que se generen efectos gravitatorios en presencia de un objeto masivo, no es sino un cuerpo negro que ocupa la totalidad del universo. Un gigantesco y tridimensional cuerpo negro que hace lo mismo que cualquier cuerpo negro: absorber todo tipo de frecuencias. Siendo el fotón el responsable de la radiación electromagnética es absorbido constantemente por espacio-tiempo. Esa es la razón de por qué el fotón se aleja invariablemente de su fuente de emisión. Ahora bien: por qué no se acumulan al lado de afuera de su fuente de emisión? Porque la materia satura el espacio con la frecuencias que emiten sus partículas componentes, de manera que el campo de absorción que genera dicho cuerpo negro sólo puede actuar en sentido opuesto a dicha saturación. Tercera pregunta: de donde obtienen su asombrosa velocidad? La densidad del cuerpo negro espacio-temporal, al que personalmente prefiero llamar campo de absorción cuántico, posee la misma densidad de masa que un agujero negro; es decir, es una especie de sopa de densidad cuántica que ocupa todo el espacio; generando un campo de una intensidad tal que absorbe a los fotones hasta acelerarlos a 300.000 Km/s. Son ellos antigravitacionales? Pues depende de cómo lo mires. En presencia de objetos como planetas o estrellas si; pero en presencia de objetos como agujeros negros no. El fotón siempre es afectado por los campos gravitacionales en más o menos medida, todo depende de la intensidad de dichos campos.

    • Hola Roberto. Creo que la respuesta es esta; El fenómeno se llama efecto fotoeléctrico y fue descrito y bien resumido por Maxwell a principio de siglo pasado. Basicamente esto movió el mundo de la f[isica de ese momento y fue una de las bases fundamentales en las cuales se apoyaría la Teor[ia de la Relatividad de Einstein. El asunto en resumidas cuentas y sin muchas fórmulas es que si imprimes energía a un cuerpo, ya sea que lo calientes, lo golpees, lo electrifiques, etc…dicha energ[ia es absorbida por lo átomos, específicamente los electrones ese de cuerpo, lo que los hace saltar de sus orbitas porque ganan energía, quedan exitados y salta a otra orbita más alejada del nucleo, pero al hacerlo emiten fotones, los fotones se irradian en forma de ondas o partículas, depende el caso. Lo hacen con una frecuencia y con una longitud que dependen de las condiciones con las cuales exitaste los electrónes que los originaron. Es como el famoso ejemplo de tirar la piedra en el lago y ves los efectos de las ondas que se producen. Es el mismo principio, pero lo que las personas no se dan cuenta es que el lago en su superficie, osea, lo que vemos es una superficie de dos dimensiones, solo se ve la onda alejarse a lo largo y lo ancho. La verdad es que en el caso de la emisión de fotones esta es irradiada en forma esféfica y constante (basicamente en todas direcciones del espacio). Sobre la segunda pregunta: ¿Por qué ellos no se acumulan al lado de afuera de su fuente de emisión?. No se acumulan su movimiento en forma de ondas no se detiene, pueden colisionar y desviarse en otras direcciones, pero no se detienen o acumulan porque su velocidad es constante, osea la de la luz. La TERCERA PREGUNTA; ¿De donde obtienen su asombrosa velocidad? si carecen de masa. La respuesta es que Tú te respondiste solo. La clave está precisamente en que carecen de masa. De hecho, una partícula con masa cero, como los son el fotón, el gluón o el todavía no confirmado gravitrón, son pura energía por tanto alcanza la velocidad máxima en nuestro universo de 4 dimensiones. (Pero no es que ganen energía y llegen hasta esa velocidad gradualmente. NO, su velocidad siempre es constante) De hecho, en las ya confirmadas ondas gravitacinales, la radiación electromagnética y la luz, viajan en el vacio exactamente a la misma velocidad; la velocidad de la luz…NINGUNA TIENEN MASA, solo son partículas mensajeras de la fuerza.

      CUARTA PREGUNTA: La verdad es que no son antigravitatorios. Lo único conocido con efectos antigravitatorios es la energía oscura, que en vez de atraer repele la materia y su mensajero el GRAVITON…sería algo as[i como una antigravedad. Pero lo aclaro mas abajo en un comentario que pegué anteriormente….si te intereza saber la respuesta exacta te recomiento que lo leas.

      Einstein lo dejó bien claro. Los Fotónes no tienen masa, por tanto la gravedad no les afecta. La gravedad solo afecta a los cuerpos que la tienen y este efecto se manifiesta como una deformación espacio-temporal. Los fotónes solo siguen la trayectoria del espacio tiempo por el cual se propagan. De ser así como propones, la teoría cuántica gravitatoria estaría resuelta porque sabríamos cómo se relaciona la gravedad con los fotones al igual que lo hace el efecto fotoeléctrico. A decir verdad los fotones ni si quiera tienen carga o una energía intrinseca, esta depende de ciertos atributos que son consecuencia de otros factores. Para que te hagas una idea de qué es realmente un fotón como una radiación de onda-partícula(depende el caso) o boson mensajero que (no genera, si no que mas bien) trasfiere energía. Es todo. Te pego un comentario que hice hace tiempo en otro lado donde trato de explicarme mejor al respecto y el por qué un fotón no tiene masa respecto a la Teoría de la Relatividad de Einstein.

      No soy físico aún ni nada, pero estudio el tema desde algún tiempo. Sencillamente en lenguaje llano te puedo decir que NO TIENEN MASA. Me explico; Puesto que la luz al estar formada por fotones viaja a la velocidad máxima solo alcanzable por energía pura al nivel mas elemental. Nada con una masa en este universo o por lo menos con tres dimensiones puede viajar más rápido que un fotón de acuerdo con la Relatividad de Einstein expresada en la fómula E=MC2. (esta nos dice que la energía pura en este caso el fotón, sería el producto de una masa acelerada a la velocidad de la luz o viceversa. “Una puede ser convertidad en la en la otra, de hecho una mepequeña cantidad de masa puede contener una gran cantidad de energía (que se disipan en forma de ondas que viajan a la velocidad de la luz…”fotones”…)(pasa todo el tiempo dentro de los colisionadores de partículas y en las explosiones nucleares)…O en caso contrario al acelerar una masa, y para que esta pueda llegar a la velocidad de la luz, requeriría de una energa infinita que como sabes es algo imposible. Puedes ver en la misma ecuación que cada vez que la masa se acerca a la velocidad de la luz la energía necesaria para acelerar se multiplica exponencialmente por dos. (M=E/C2). Es decir, nunca llegarías a tener toda la energía necesaria porque mientras mas cercano a la velocidad de la luz estés, más energía necesitarás… Por tanto los fotones NO TIENEN MASA y por eso alcanzan la velocidad de la luz o la fotónica, que es lo mismo… De hecho las ondas electromagnéticas no tienen masa, la gravedad no les afecta y es un fenómeno que viaja a la velocidad de la luz, su partícula mensajera es el fotón. Simplemente imagínalos como objetos de dos dimensiones (existen como entidad en el tiempo y tienen existencia en otra dimensión adicional).

      • Parece que hablas de la radiación electromagnética y la luz como si no fueran la misma cosa … Por otra parte espero que la RAE no haya aceptado escribir excitar sin ce, aunque últimamente ya cualquier cosa.

  32. Muy buen artículo! Llevo un buen rato releyendo y me han asaltado unas dudas:
    (1) (E/c)^2 qué significado físico tiene?
    (2) El momento «p» se calcula usando la masa relativista? Si se usa la masa en reposo en lugar de la relativista, porqué?
    (3) m^2*c^2 para la masa en reposo, qué significado físico tiene? Es el momento de la masa en reposo a la velocidad de la luz, al cuadrado?
    (4) Visto desde otro ángulo, despejando (E/c)^2, este valor es la suma de dos momentos cuadrados calculados para diferente masa?
    (5) De ser la misma masa para calcular los momentos, por qué?
    Gracias por el esfuerzo de acercar la relatividad a nosotros los legos. De ahora en adelante seré visitante asiduo de este espacio.

  33. Hola Domingo. Aunque ya tiene algún tiempo tu comentario, me gustaría decirte que es muy interesante lo que planteas aquí:

    «debemos suponer que la masa es una especie de aglutinado de impulsos que permanecen tirando tenso en todas direcciones y sentidos, lo cual hace que el sistema permanezca sin movimiento. Si aportamos ahora una cantidad de impulso en una determinada dirección estaremos desequilibrando el sistema el cual ahora habrá incrementado su masa de impulso, o sea su masa, y además estará en movimiento».

    De hecho mi teoría establece algo muy parecido. Aunque a esa «especie de aglutinado de impulsos que permanecen tirando tenso en todas direcciones y sentido», yo lo llamo Campo. El mismo campo por cierto, que conocemos con el nombre de Campo Unificado y a través del cual se genera toda interacción entre partículas y objetos gravitacionales. Las cuales se designan con el nombre de débil, fuerte, electromagnética y gravitatoria, como ya sabes. Ahora bien, existe una relación estrecha entre masa y espacio-tiempo que mucha gente desconoce. Ya que todo objeto que ocupa espacio tiene masa.
    Esos impulsos que tu llamas, se generan por medio de la interacción que se produce entre las partículas que componen la materia con dicho
    campo. De manera que efectivamente, un sistema permanece sin movimiento porque las interacciones que se producen en dicho sistema están en equilibrio con el campo. En el caso de un átomo, al estar compuesto por partículas menores, que a su vez interaccionan con dicho campo, se genera una interacción con el mismo que actúa hacia dentro y hacia afuera del mismo. Con lo cual el átomo permanece en equilibrio y por tanto no experimenta movimiento. (Esto es válido para cualquier partícula con masa u objeto) Sin embargo el fotón es un caso especial. El fotón no ocupa espacio, ya que no está compuesto por partículas menores; y la mejor forma de demostrar esto es precisamente porque el campo, al no poder actuar en su interior, lo que hace es tirar hacia afuera del mismo y acelerarlo hasta la velocidad de la luz. Por eso el fotón no puede tener masa, ya que no ocupa espacio.
    Un cordial saludo.

  34. Pingback: La simple genialidad de Yukawa | Cuentos Cuánticos

  35. El fotón no tendría masa en reposo (que por cierto un fotón nunca podría estar en reposo) de acuerdo. Por otra parte, puede decir que si tiene masa en movimiento ¿cómo se calcula la masa de un fotón concreto? Gracias

    • Hola Campoalvar. La respuesta mas sencilla es que …Los fotones no se originan como la transformación o expulsión de la masa de un electrón u otra partícula, ya que la masa de un electrón es invariante. El fotón ya sea como onda o partíula, el gravitón el Higgs son bosones, su masa en movimiento es cero. Por tienen la cualidad de viajar a la velocidad máxima en nuestro universo, osea la velocidad de la luz, pero no es que vallan convirtiendo algún tipo de masa propia en energía para poder alcanzar la verlocidad de la luz como lo propone Einstein. Su velocidad siempre es constante, desde su emisión hasta su absorción. Siendo de esa manera, como dice Einstein, la energía necesaria para acelerar una masa a la velocidad de la luz, aunque fuere tan pequeña como la de un fotón, sería infinita. SENCILLAMENTE NO SE PUEDE CALCULAR NINGUN TIPO DE MASA EN UN FOT[ON, PORQUE NO LA TIENE. Sin embargo existe la llamada masa relativista de un cuya fórmula es h/λc o E/c2, pero es siemplemente la equivalencia energía a unidades de masa con fines de cálculo. Pero como dije antes no es que sea la masa propiamente dicha de un fotón.

    • Manuel Sánchez Carrilero

      Al final pondré un comentario razonado que responda, en la medida de lo posible, las dudas interesantes de algunos usuarios como es tu caso.
      Gracias

    • Manuel Sánchez Carrilero

      La energía relativista viene dada por la expresión:
      m c^2 = mo c^2 + T (1)
      siendo E = m c^2 la energía total relativista, donde m es la masa de la partícula.
      Dicha energía se descompone en un sumando denominado energía en reposo, en función de la llamada masa en reposo mo, que es una constante característica de cada partícula y sería el valor de m cuando v=0, es decir cuando dicha partícula está en reposo respecto al observador.
      El segundo sumando T es la energía cinética relativista.
      Notar aquí la primera falacia en la que incurren algunos usuarios cuando dicen que la expresión E=mc^2 solamente es una fórmula de equivalencia entre una energía y una masa, sin reparar en que la expresión (1) es obtenida por un proceso de integración fuerza desplazamiento, teniendo en cuenta la (2).
      Todo esto es consistente con la expresión relativista de la masa en función de las velocidades dada por
      m=mo/SQR(1-(v/c)^2) (2)
      que nos dice que la masa m en movimiento a la velocidad v es mayor que la masa en reposo mo.
      ¿Qué sucede si se trata del fotón? Como la masa en reposo para el fotón es mo=0, la primera ecuación (1) dará
      mc^2 = T
      es decir la energía total es toda cinética, y dicho fotón puede moverse solo con la velocidad de la luz y nunca puede estar en reposo en un sistema inercial.
      La segunda ecuación (2), poniendo mo=0 y también v=c, dará una indeterminación del tipo 0/0.
      Antes de seguir relato la segunda falacia: La velocidad de la luz c es siempre constante cualquiera que sea el observador. Debe ser reemplazada por: La velocidad de la luz c es siempre la misma para todos los observadores inerciales.
      Veamos la vidriosa cuestión de que la masa del fotón es siempre cero: me remito a la página 460, del libro La Gran Ilusión, en el capítulo: dentro y fuera del ascensor, donde la voz autorizada de Stephen Hawking, dice “Un haz de luz posee energía y la energía tiene masa…” Se está refiriendo al famoso experimento mental del ascensor de Einstein, en que la luz adquiere curvatura tanto en un campo gravitatorio, como en un ascensor acelerado, todo basado en la equivalencia exacta entre masa gravitatoria y masa inerte (en este caso del fotón).
      Otro relato, página 514, Física Teórica, vol. 2, donde J. Cabrera y Felipe dice “…tengamos en cuenta que la masa correspondiente a un fotón es hf/(c^2)…” Se está refiriendo al choque de un fotón de frecuencia f con otra partícula (electrón), como sucede en el efecto Compton.
      Conclusiones: la luz, el fotón, tiene masa NO NULA que se obtiene al despejar m de la igualdad de energías, relativista y de Planck, según la relación mc^2 = hf.
      Por otro lado, vuelvo a reafirmarme en que lo que sí es cero es la masa mo del fotón, es decir la masa en reposo. Es más si tratáramos de calcular la masa por la relación relativista en función de las velocidades vista arriba (2) obtendríamos 0/0 como ya dijimos, porque esa fórmula relativista es válida para cualquier partícula con v<c.

  36. Hay una forma de explicar que es la masa que usa conceptos mecánicos simples. Como dice la ecuación m = P/c la masa es una cantidad de impulso dividido por c. La ecuación P = mc nos está diciendo cuanta cantidad de impulso constituye o contiene la masa, es decir, 1c Ns por kilogramo de masa. ¿puede una masa en reposo tener impulso? Parece que no pues P = mv y v es cero para la masa en reposo. Sin embrago la ecuación está ahí insitiendo en lo mismo.
    De modo que se lo podemos creer, para ello debemos suponer que la masa es una especie de aglutinado de impulsos que permanecen tirando tenso en todas direcciones y sentidos, lo cual hace que el sistema permanezca sin movimiento. Si aportamos ahora una cantidad de impulso en una determinada dirección estaremos desequilibrando el sitema el cual ahora habrá incrementado su masa de impulso, o sea su masa, y además estará en movimiento.
    Lo cierto es que partiendo de esta idea se puede explicar toda la relatividad especial, la masa y la inercia, es decir, las cuentas cuadran perfectamente, haciendo de la masa un almacen de impulsos y de la relatividad especial una forma de gestionar las entradas y salidas de impulsos de dicho almacén.

  37. Pingback: No solo del Higgs vive la masa | Cuentos Cuánticos

  38. No persigo nada.

    Simplemente que el foton tiene masa o no dependiendo de como definas la masa. Tu no has demostrado que el foton no tenga masa, lo que has demostrado es que no se puede definir la energía en reposo en el foton.

    La definición de masa dada por E=mc^2 es tan coherente como la que tu has utilizado explicitamente y con esa definición el foton tiene masa.

    Demostrar es deducir de premisas previas, tu solo has definido la masa de una de las posibles maneras de hacerlo pero no has demostrado.

    Gracias por tu atención.

    • La masa solo tiene una definición, en ultima instancia es el autovalor de un Casimir del grupo de Poincaré que es bajo el cual catalogamos a las partículas elementales.

      Tu definición, y tus multiplicaciones de masas y derivadas no pueden generalizarse a vectores nulos. Eso está explicado en todos los libros de relatividad.

      Por otra parte, ya que el argumento relativista no te convence, genial, el argumento gauge es mucho más potente bajo mi punto de vista. Y ese, claramente, nos dice que la masa del fotón tiene que ser cero (y encima la masa que aparece en los términos de masa de los lagrangianos de la teoría son compatibles con la definición que se usa en la sección previa).

      Un saludo.

      • Antes que nada, quiero decir que la explicación esta muy bien trabajada, felicidades. En cuanto a qué es la masa … creo que hay cierta confusión. La masa no se puede medir sin campo, es decir, experimentamos la masa de las cosas debido a su campo gravitatorio. Las simetrías de Gauge explican las cosas desde un punto de vista muy limitado, y no encaja con la dualidad de la radiación electromagnética. El fotón sería la partícula que origina el flujo y el campo, los cuales no necesitan de masa para ser; pero al fotón es dificil seguirle la pista. La pregunta que debemos hacernos es si el fotón curva su trayectoria, al pasar por un campo de gravedad muy potente, será causa de que tiene masa y por tanto es atraído por ese campo, o es por otra extraña razón.

        En cuanto a que no se puede detener un fotón, la cosa no esta tan clara. Si el fotón es la partícula que genera la interacción de la luz ( y otros tipos de radiaciones electromagnéticas ) ¿cómo se explica que se haya conseguido retener ( detener o retardar ) la luz durante un minuto en un «espejo»? http://prl.aps.org/abstract/PRL/v111/i3/e033601

        Según mi teoría el fotón es una partícula formada por dos bosones ( casi con seguridad Higgs ) los cuales orbitan a la velocidad de la luz y a una distancia tal que solo percibimos los efectos que producen a su paso por el espacio; la radiación electromagnética. Mi teoría explica los fenómenos sin tener en cuenta el tiempo, lo cual alarga la explicación, pero de un modo análogo en el siguiente link se puede entender con la dimensión temporal. https://sites.google.com/site/teoriatiempoespacio/los-fotones

        • Permíteme que te conteste por bloques:

          La masa no se puede medir sin campo, es decir, experimentamos la masa de las cosas debido a su campo gravitatorio.

          La masa se puede medir por experimentos que involucren inercia. La masa, además de ser la carga gravitatoria, mide la inercia de los sistemas. Así pues, en el contexto en el que estamos en esta entrada estamos considerando la masa inercial y no la gravitatoria puesto que en las teorías de las interacciones no gravitatorias, la gravedad no está presente y aún así podemos definir la masa.

          Las simetrías de Gauge explican las cosas desde un punto de vista muy limitado, y no encaja con la dualidad de la radiación electromagnética

          Las simetrías gauge son la mejor explicación que tenemos sobre la mesa de las interacciones no gravitatorias. Su validez se refuerza en cada experimento de altas energía que se realiza. Su origen y su contexto es amplio y está enraizado en la geometrización de las interacciones a nivel cuántico. Las teorías gauge son importantes porque todas las interacciones verifican una ley análoga a la ley de Gauss del electromagnetismo y esta ley obliga a imponer la simetría gauge correspondiente. De hecho, no hay alternativas conocidas y consistentes a la simetría gauge.

          De no ser correcta la hipótesis de la simetría gauge sería muy difícil explicar la presencia de bosones mensajeros como el fotón, el W, el Z o los gluones ya que estos son partículas de origen gauge.

          No confundamos aquí el hecho de que diga que son partículas gauge con la idea de que estamos obligando a la naturaleza a seguir nuestras teorías. El caso es que si estas partículas no concordaran con las teorías gauge estas últimas serían inútiles y tendríamos que rechazarlas.

          El fotón sería la partícula que origina el flujo y el campo, los cuales no necesitan de masa para ser; pero al fotón es dificil seguirle la pista. La pregunta que debemos hacernos es si el fotón curva su trayectoria, al pasar por un campo de gravedad muy potente, será causa de que tiene masa y por tanto es atraído por ese campo, o es por otra extraña razón.

          Que la luz se curva en presencia de campos gravitatorios es algo conocido, muy bien conocido, hay muchos experimentos y observaciones que lo confirman (por ejemplo, las lentes gravitacionales que son magníficos test observacionales para la relatividad general).

          Hemos descubierto, relatividad general, que la fuente de la gravedad no es la masa sino la energía. Además sabemos que masa y energía son aspectos distintos de una misma entidad. Así que algo puede tener energía pero no masa y eso no hace que no sienta el campo gravitatorio, no siendo este último más que la manifestación de la geometría dinámica del espaciotiempo respondiendo a la presencia de otros campos.

          En cuanto a que no se puede detener un fotón, la cosa no esta tan clara. Si el fotón es la partícula que genera la interacción de la luz ( y otros tipos de radiaciones electromagnéticas ) ¿cómo se explica que se haya conseguido retener ( detener o retardar ) la luz durante un minuto en un “espejo”? http://prl.aps.org/abstract/PRL/v111/i3/e033601

          Hay que tener cuidado con la interpretación de estas noticias y estos artículos. Un fotón, en el vacío no se puede detener porque está obligado a viajar a la velocidad de la luz. Así es como lo dictamina la teoría de la relatividad.

          Pero para fotones en medios distintos al vacío no hay ningún impedimento en «frenarlos», «pararlos», «detenerlos» o «congerlarlos». Aún así hay que tener cuidado y hablar con propiedad, lo que se suele hacer es congelar el estado cuántico del fotón en un medio material, cosa perfectamente permitida por la relatividad, o modificar su velocidad de fase o de grupo. Eso no contradice lo dicho en la entrada.

          Según mi teoría el fotón es una partícula formada por dos bosones ( casi con seguridad Higgs ) los cuales orbitan a la velocidad de la luz y a una distancia tal que solo percibimos los efectos que producen a su paso por el espacio; la radiación electromagnética. Mi teoría explica los fenómenos sin tener en cuenta el tiempo, lo cual alarga la explicación, pero de un modo análogo en el siguiente link se puede entender con la dimensión temporal. https://sites.google.com/site/teoriatiempoespacio/los-fotones

          El fotón tiene espín (helicidad) 1 y el Higgs es de espín 0. No es posible combinar dos bosones de espín nulo para formar uno de espín 1. Eso violaría una montaña de leyes físicas comprobadas experimentalmente hasta la saciedad.

          Si tienes algún artículo científico publicado en el que pueda leer tu propuesta y en el que vea el esquema teórico y la estructura matemática de tu idea podría echarle un vistazo. Pero por lo que me has comentado tengo que decirte que la idea no tiene sentido en física y que no es posible implementarla matemáticamente.

          Un saludo

          • Enrique gracias por tus aclaraciones, me da que pensar algunas cosas, aunque otras me confunden un poco.

            Dices que «Hemos descubierto, relatividad general, que la fuente de la gravedad no es la masa sino la energía.» Esto es un avance impresionante ¿puedes explicarme esto o darme una referencia que lo demuestre?

            Tambien dices que «No es posible combinar dos bosones de espín nulo para formar uno de espín 1. Eso violaría una montaña de leyes físicas comprobadas experimentalmente hasta la saciedad.» En realidad si es posible, de hecho es la única manera y en ello se sustenta la teoría de Higgs y en gran parte la mía. Realmente se denominan incorrectamente bosones a las partículas Higgs, y los bosones realmente no son la combinación de ellos sino el residuo que se obtiene de su combinación. Es largo de explicar, pero para hacernos una idea, supon que los higgs giran en torno a sí mismos en el mismo plano que orbitan pero en sentido contrario. El eje de su órbita producirá una «helice» ( no me gusta el simil pero bueno ) que dependiendo de cuantas veces cambie el plano de giro en cada vuelta completa podrá ser el spin 1 o 1/2. Dependiendo del numero de higgs que formen un fermión, en forma de nube de higgs, obtendremos en cada par liberado un higgs y un fotón. ¿De que manera esto puede ser incorrecto?

            En cuanto a la simetria Gauge estoy de acuerdo en que es la mejor opción hasta ahora, pero no es la única y el hecho de que tengamos que deducir las cosas por los campos de fuerza nos limita a suponer la existencia de las partilucas pero no nos da una información directa de estas. Pero en este tema mejor no entrar porque entonces podemos dudar de hasta nuestra existencia 😉

            Gracias y un saludo.

            • Dices que “Hemos descubierto, relatividad general, que la fuente de la gravedad no es la masa sino la energía.” Esto es un avance impresionante ¿puedes explicarme esto o darme una referencia que lo demuestre?

              En cualquier texto de Relatividad General se demuestra esto. Esencialmente las ecuaciones de Einstein tienen la forma:

              G=8πT

              G = es el tensor de Einstein que depende de la métrica del espaciotiempo y sus derivadas.
              T= Tensor de Momento-Energía. En él se introduce toda la energía de los campos no gravitatorios presentes en una región de espaciotiempo. Así que no solo es la masa lo que produce gravedad, la fuente gravitatoria es la energía en si misma.

              Tambien dices que “No es posible combinar dos bosones de espín nulo para formar uno de espín 1. Eso violaría una montaña de leyes físicas comprobadas experimentalmente hasta la saciedad.” En realidad si es posible, de hecho es la única manera y en ello se sustenta la teoría de Higgs y en gran parte la mía. Realmente se denominan incorrectamente bosones a las partículas Higgs, y los bosones realmente no son la combinación de ellos sino el residuo que se obtiene de su combinación. Es largo de explicar, pero para hacernos una idea, supon que los higgs giran en torno a sí mismos en el mismo plano que orbitan pero en sentido contrario. El eje de su órbita producirá una “helice” ( no me gusta el simil pero bueno ) que dependiendo de cuantas veces cambie el plano de giro en cada vuelta completa podrá ser el spin 1 o 1/2. Dependiendo del numero de higgs que formen un fermión, en forma de nube de higgs, obtendremos en cada par liberado un higgs y un fotón. ¿De que manera esto puede ser incorrecto?

              No, no es posible.

              La combinación de espines es algo bien conocido y rígido. No se puede modificar, así que si uno tiene dos espines 0 (que indicaría el módulo de un vector en un espacio complejo) no se puede conseguir espín 1.

              Esto se puede encontrar en cualquier texto de algebras de Lie o de mecánica cuántica en el que traten el tema de composición de momento angular. Por ejemplo en el capítulo 14 de estas notas: http://www.physics.sfsu.edu/~greensit/book.pdf

              Otra cuestión es que a partir de bosones no se pueden obtener fermiones, por las reglas del momento angular, que es una cantidad conservada. Recuerda que el espín sigue esas mismas reglas.

              Además, dificil será combinar dos bichos con masa para generar un bicho sin masa.

              En cuanto a la simetria Gauge estoy de acuerdo en que es la mejor opción hasta ahora, pero no es la única y el hecho de que tengamos que deducir las cosas por los campos de fuerza nos limita a suponer la existencia de las partilucas pero no nos da una información directa de estas. Pero en este tema mejor no entrar porque entonces podemos dudar de hasta nuestra existencia

              Si no es la única, ¿cuál es la alternativa?

              Las cosas a un nivel fundamental no se determinan por campos de fuerzas sino por potenciales gauge, la diferencia es notable. Y dada una simetría gauge local, de forma inmediata, se predice la existencia de las partículas portadoras de una interacción. Así que hay poco donde rascar.

              De hecho, las únicas teórias cuánticas consistentes (libres de infinitos) para las interacciones son las teorías gauge. Cualquier otra versión propuesta está plagada de infinitos que no se pueden domar, esto hace que la propuesta sea poco útil.

              E insisto, si tienes una teoría me gustaría ver su desarrollo. ¿Qué objetos la definen? ¿Qué matemática hay detrás de ella? ¿Qué se puede calcular con ella? ¿Cómo reproduce los hechos experimentales conocidos hasta la fecha? ¿Qué nuevos fenómenos predice que se puedan comprobar experimentalmente?

              Un saludo

              • Enrique creo que el tensor tensión-energía no se refiere a eso, la energía de la materia a nivel subatómico se debe tratar con cierto respeto, no podemos aplicar leyes macrofísicas a esta escala. Por otra parte, aunque el experimento mental de Einstein fue muy productivo hoy en día sabemos que tenemos que buscar alternativas a las paradojas a las que nos conduce. Me parece que se nos queda pequeño este lugar para todo lo que tenemos que debatir. Si te parece bien, podemos continuar en un hilo a parte, creo que la discusión lo merece.

                • Querido Iñigo, el tensor energía-momento se refiere justamente a eso. Está extraído de la acción que define la teoría y esta a su vez viene determinada fuertemente por las simetrías de la naturaleza para cada interacción, campo y partículas que estudiamos. Su validez ha sido confirmada experimentalmente en los últimos 300 años en cada experimento realizado en física hasta la fecha.

                  La energía, ya sea a nivel cuántico, clásico, macro o microscópico es una cantidad bien definida y entendemos bien su origen dentro de las ecuaciones con las que describimos la naturaleza. Además, insisto, la conocemos tan bien que todos nuestros experimentos hasta la fecha validan lo que sabemos de lla.

                  Respecto al experimento mental de Einstein, no sé a qué te refieres y tampoco a qué paradojas haces referencia. La Relatividad General ha superado todos los test experimentales y observacionales que se le han impuesto, algunos de ellos creados para mostrar que la teoría era errónea.

                  Podemos seguir donde quieras, pero aún no me has dicho donde puedo ver el desarrollo teórico-conceptual de tu teória.

                  Un saludo.

    • Pero… yo tenía entendido que E=mc2 no significa que todo lo que tiene energía tiene masa sino que la masa puede convertirse en energía y la energía en masa o que la masa es una forma muy condensada de energía.

  39. Ok, la masa en reposo del fotón es cero. Ahora, la masa en movimiento es la masa en reposo dividido «tau»…

    O sea que la masa de un fotón que se mueve a la velocidad de la luz sería 0/0?? Qué nos dice esto, que esa fórmula no nos sirve para calcular la masa del fotón en movimiento? O en movimiento tampoco tiene masa y toda su energía pertenece a su campo electromagnético? Supuestamente el efecto fotoeléctrico parecía demostrar que el fotón se comportaba como una partícula con masa.

    • Manuel Sánchez Carrilero

      La energía relativista viene dada por la expresión:
      m c^2 = mo c^2 + T (1)
      siendo E = m c^2 la energía total relativista, donde m es la masa de la partícula.
      Dicha energía se descompone en un sumando denominado energía en reposo, en función de la llamada masa en reposo mo, que es una constante característica de cada partícula y sería el valor de m cuando v=0, es decir cuando dicha partícula está en reposo respecto al observador.
      El segundo sumando T es la energía cinética relativista.
      Notar aquí la primera falacia en la que incurren algunos usuarios cuando dicen que la expresión E=mc^2 solamente es una fórmula de equivalencia entre una energía y una masa, sin reparar en que la expresión (1) es obtenida por un proceso de integración fuerza desplazamiento, teniendo en cuenta la (2).
      Todo esto es consistente con la expresión relativista de la masa en función de las velocidades dada por
      m=mo/SQR(1-(v/c)^2) (2)
      que nos dice que la masa m en movimiento a la velocidad v es mayor que la masa en reposo mo.
      ¿Qué sucede si se trata del fotón? Como la masa en reposo para el fotón es mo=0, la primera ecuación (1) dará
      mc^2 = T
      es decir la energía total es toda cinética, y dicho fotón puede moverse solo con la velocidad de la luz y nunca puede estar en reposo en un sistema inercial.
      La segunda ecuación (2), poniendo mo=0 y también v=c, dará una indeterminación del tipo 0/0.
      Antes de seguir relato la segunda falacia: La velocidad de la luz c es siempre constante cualquiera que sea el observador. Debe ser reemplazada por: La velocidad de la luz c es siempre la misma para todos los observadores inerciales.
      Veamos la vidriosa cuestión de que la masa del fotón es siempre cero: me remito a la página 460, del libro La Gran Ilusión, en el capítulo: dentro y fuera del ascensor, donde la voz autorizada de Stephen Hawking, dice “Un haz de luz posee energía y la energía tiene masa…” Se está refiriendo al famoso experimento mental del ascensor de Einstein, en que la luz adquiere curvatura tanto en un campo gravitatorio, como en un ascensor acelerado, todo basado en la equivalencia exacta entre masa gravitatoria y masa inerte (en este caso del fotón).
      Otro relato, página 514, Física Teórica, vol. 2, donde J. Cabrera y Felipe dice “…tengamos en cuenta que la masa correspondiente a un fotón es hf/(c^2)…” Se está refiriendo al choque de un fotón de frecuencia f con otra partícula (electrón), como sucede en el efecto Compton.
      Conclusiones: la luz, el fotón, tiene masa NO NULA que se obtiene al despejar m de la igualdad de energías, relativista y de Planck, según la relación mc^2 = hf.
      Por otro lado, vuelvo a reafirmarme en que lo que sí es cero es la masa mo del fotón, es decir la masa en reposo. Es más si tratáramos de calcular la masa por la relación relativista en función de las velocidades vista arriba (2) obtendríamos 0/0 como ya dijimos, porque esa fórmula relativista es válida para cualquier partícula con v<c.

  40. Tu estas aplicando para un fotón, algo que se mueve a la velocidad c, una expresión que solo puedes aplicar a una partícula que se mueve a V<c, y eso no tiene sentido hacerlo así por eso mismo que indicas: "el tiempo propio de algo que se mueve a "c" es cero".

    El vector [TEX]\left( c,\vec{c}\right)[TEX] es totalmente legitimo y se obtiene diferenciando el vector de posición, en la superficie del cono de luz, con respecto al tiempo del observador. Es invariante en modulo y este vale cero lo que indica que se encuentra en la superficie del cono de luz.

    • Lo siento pero tu razonamiento es erróneo y estás mezclando muchos conceptos. Sintiéndolo mucho, no llevas razón.

      Además no veo qué persigues con estos argumentos, ¿decir que el fotón si tiene masa?

  41. Decir que algo no tiene sentido sin indicar y demostrar donde está el fallo es poco útil.
     
    Puedo intentar demostrar la falsedad o acierto de algo «con sentido» pero me es imposible demostrar que algo que «no tiene sentido» no tiene sentido.
     
    Un fotón se mueve a c, y se mueve a c para cualquier observador. Es decir, no puede estar en resposo para ningún observador con lo cual tiene mucho sentido hablar de reposo y hablar de la masa nula del fotón en reposo. Es la terminología física estándar
     
    Perdona, pero no le encuentro sentido a que algo que no puede estar en reposo pueda tener algo definido en el reposo. Pero te reconozco que no puedo demostrarte que no tiene sentido.
     
    El momento es algo más que la masa, de hecho en ondas, donde no hay transporte de materia también se puede definir el momento. Así como en diversos campos como electromagnético o gravitatorio. La definición p=mv solo es válido para cuerpos con masa y velocidades pequeñas comparadas con c. La masa no es esa relación porque según la relatividad, y la clasificación de representaciones del grupo de Poincaré es el módulo del cuadrimomento y no sus componentes.
     
    Estamos hablando de relatividad especial.
    Para una particula que se mueve a una velocidad menor que «c» se define el tetravector velocidad como \left( \gamma c,\gamma \vec{v}\right) que tiene modulo c^2, y de hay el tetravetor momento multiplicando por una constante , sale

    {m}_{0 }\left( \gamma c,\gamma \vec{v}\right)

    que tiene modulo {{m}_{0 }}^{2 } {c}^{2 }, y donde esa cosntante es la masa en reposo. No es dificil darse cuenta que definiendo

    \frac{E}{c }= {m}_{0 }\gamma c
    \vec{p}={m}_{ 0}\gamma \vec{v}

    el modulo del tetravector momento cumple aquello de:

    {E}^{2 }= {{m}_{ o}}^{2 }{c}^{4 } + {p}^{ 2}{c}^{2 }

    Si consideramos ahora una particula con una masa en reposo muy muy pequeña con respecto a su energia relativista total, o es dificil ver que se cumple:

    {E}^{2 }\approx {p}^{ 2}{c}^{2 }

    que es la expresion que como se vera mas adelante cumple un foton. Pero un foton no es una particula con una masa en reposo muy, muy, muy pequeñita y una velocidad muy, muy, muy grande como parace deducirse de tu articulo.
     
    Ese razonamiento no es corecto, la derivación del cuadrimomento no es así como se puede ver en cualquier texto de relatividad.
    Lo que queda es:
    {E}^{2 }= {{m}_{ o}}^{2 }{c}^{4 } + {p}^{ 2}{c}^{2 }
    Si esta expresión, por los motivos dichos en la entrada, da 0, es porque m^2c^4=0 y c no es cero, así que nos queda que m=0.

    El foton no es una particula con una masa en reposo muy, muy, muy pequeñita. Esa expresion no es valida para algo que se mueve a «c», solamente es valida para algo que se mueve a «v<c". Para hayar la expresion que cumple el foton debemos partir del tetarvector velocidad dado por

    \left( c,\vec{c}\right)

    cuyo modulo es «cero»;, evidentemente. Multiplicamos por una constante «m»; y queda:

    \left(m c, m \vec{c}\right)

    cuyo modulo cumple la ralacion:

    {E}^{2 }- {p}^{ 2}{c}^{2 }=0

    si mas que asociar \frac{E}{c }= mc y \vec{p}=m \vec{c}. Se observa que la cosntante «m»; puede ser cualquiera, es decir la masa de foton puede tener cualquier valor. Se observa tambien que es la misma ecuacion que cumplen las particulas con muy poquita masa y mucha velocidad, pero el foton no es una particula con muy poquita masa y mucha velocidad, el foton es algo que no puede estar en reposo y por tanto no puede tener definada una masa en reposo.

    • No hay ningún vector relativista que cumpla (c,\vec{c}) para los fotones tampoco, la razón es simple, para definir la 4-velocidad hay que derivar respecto al tiempo propio, y resulta que el fotón no tiene definido el tiempo propio. Lo siento pero tu explicación no es consistente con la relatividad especial (puedes ver la demostración en cualquier texto serio que la trate).

      Pero además, tienes todo un conjunto de razones por las que el fotón no tiene masa, la más potente la simetría gauge del electromagnetismo.

  42. Hola, yo siempre había pensado que los intermediarios de la interacción fuerte eran los gluones, que no tienen masa, pero que la interacción tiene un alcance finito porque los gluones interaccionan entre ellos… no así los fotones.

    O me han contado un cuento (cuántico)?

    Saludos y felicidades por este magnífico blog!!

    • Lo de los gluones todavía es un tema abierto que se espera solucionar en breve. Pero sí, la teoría establece que no tienen masa pero si que portan carga fuerte, lo que los hace interactúar entre ellos produciendo el efecto de corto alcance de la interacción fuerte.

  43. No me parece muy convincente el razonamiento que se presenta para demostrar que le foton no tiene masa. Mas bien creo que se demuestra que el foton no tiene masa en reposo, lo cual, con la definicion de reposo pienso que es un sin sentido.

    La masa en general se define con la inercia al cambio de movimiento. En la mecánica clásica este cambio esta asociado al momento y la masa se deriva de este simplemente dividiendo por la velocidad, en este caso «c». Asi que, según esto, el foton tiene una masa dada por E/c^2 = pc/c^2

    Es cierto que la masa definida tal como se define en el articulo tiene la innegable ventaja de ser un invariante relativista, lo mismo que es un invariante relativista todo aquello que puedas referir «al reposo», pero como indico no tiene ningún sentido decir que el foton en reposo tiene masa cero, lo mismo que no tiene ningún sentido decir que un electron tiene masa imaginaria a una velocidad 2c.

    Evidentemente todo el razonamiento es muy inconsistente. Para verlo solo hay que poner la expresión del cuatrivector velocidad y multiplicar por «m», la masa del foton::

    s^2=(ct)^2-x^2=c^2-(x/t)^2

    para el foton, algo que se mueve a la velocidad «c», la expresión queda:

    0=c^2-(c^2)

    que multiplicada por la masa de foton queda:

    m·0=mc^2-mc^2 = E-(mc)·c= E- p·c

    que evidentemente se verifica para cualquier m, es decir, que el foton puede tener cualquier masa y no como dice el articulo que su masa solo puede ser cero.
    (me molesta mucho que se diga que la masa del foton es cero, pero me molesta mucho mas que encima lo demuestren)

    • No me parece muy convincente el razonamiento que se presenta para demostrar que le foton no tiene masa. Mas bien creo que se demuestra que el foton no tiene masa en reposo, lo cual, con la definicion de reposo pienso que es un sin sentido.

      Un fotón se mueve a c, y se mueve a c para cualquier observador. Es decir, no puede estar en resposo para ningún observador con lo cual tiene mucho sentido hablar de reposo y hablar de la masa nula del fotón en reposo. Es la terminología física estándar.

      La masa en general se define con la inercia al cambio de movimiento. En la mecánica clásica este cambio esta asociado al momento y la masa se deriva de este simplemente dividiendo por la velocidad, en este caso “c”. Asi que, según esto, el foton tiene una masa dada por E/c^2 = pc/c^2

      El momento es algo más que la masa, de hecho en ondas, donde no hay transporte de materia también se puede definir el momento. Así como en diversos campos como electromagnético o gravitatorio. La definición p=mv solo es válido para cuerpos con masa y velocidades pequeñas comparadas con c. La masa no es esa relación porque según la relatividad, y la clasificación de representaciones del grupo de Poincaré es el módulo del cuadrimomento y no sus componentes.

      Es cierto que la masa definida tal como se define en el articulo tiene la innegable ventaja de ser un invariante relativista, lo mismo que es un invariante relativista todo aquello que puedas referir “al reposo”, pero como indico no tiene ningún sentido decir que el foton en reposo tiene masa cero, lo mismo que no tiene ningún sentido decir que un electron tiene masa imaginaria a una velocidad 2c.

      Decir que algo no tiene sentido sin indicar y demostrar donde está el fallo es poco útil.

      Evidentemente todo el razonamiento es muy inconsistente. Para verlo solo hay que poner la expresión del cuatrivector velocidad y multiplicar por “m”, la masa del foton::

      s^2=(ct)^2-x^2=c^2-(x/t)^2

      para el foton, algo que se mueve a la velocidad “c”, la expresión queda:

      0=c^2-(c^2)

      que multiplicada por la masa de foton queda:

      m·0=mc^2-mc^2 = E-(mc)·c= E- p·c

      que evidentemente se verifica para cualquier m, es decir, que el foton puede tener cualquier masa y no como dice el articulo que su masa solo puede ser cero.

      Ese razonamiento no es corecto, la derivación del cuadrimomento no es así como se puede ver en cualquier texto de relatividad.

      Lo que queda es:

      E^2-p^2c^2=m^2c^4

      Si esta expresión, por los motivos dichos en la entrada, da 0, es porque m^2c^4=0 y c no es cero, así que nos queda que m=0.

      Pero bueno, si no te convence el argumento relativista, tenemos más, el gauge es bastante potente. Si el fotón tiene masa la carga eléctrica no se conserva y el electromagnetismo no funcionaría como conocemos.

  44. Pingback: Bitacoras.com

  45. Pingback: El fotón y la masa | Carlos E. Martínez R.

  46. Pingback: El fotón y la masa | Maths and else | Sc...

  47. Pingback: El fotón y la masa | Ciencia-Físi...

  48. Magnífica entrada CC, pero de un tirón no me la puedo enguyir, ahora creo que estoy claro del porque el fotón no tiene masa, ya que al principio me cuestionaba la misma pregunta de que si tiene energía debería tener masa, por la ecuación aquella.

  49. pregunta. si el foton se representa a través de la expresion E=pc, dejando de esta forma explicito que el foton no tiene masa, que me dice el momento pues ya que el momento se puede expresar por medio de la masa y la velocidad (mv) que papel cumple la masa en este caso

    • Es que en ondas, que también llevan momento este se expresa en función del número de ondas. La expresión mv es válida para cuerpo con masa m y velocidad v. En relatividad es algo más complicado y en ondas lo que hemos dicho.

  50. Un post cojonudo! Felicidades!!

  51. Alfonso Garcia

    La explicacion relativista no me vale. No explicas porque el cuadrimomento vale cero!

    • Creí que estaba claro del contexto. Si estamos estudiando partículas que se mueven a la velocidad de la luz entonces todos sus vectores han de estar sobre el cono de luz, lo que se llaman vectores nulos por razones obvias, y por extensión su módulo tiene que ser 0.

      Gracias por el aviso.

      • Alfonso Garcia

        Entonces para una particula masiva el modulo de cualquier cuadrivector debe ser la masa al cuadrado?

        • No, solo el cuadrimomento dará lugar a la masa. Porque es el que contiene la información «dinámica» de la partícula. Pero si el vector es nulo, aunque seal el cuadrimomento, su módulo es 0 sin serlo sus componentes.

          • Alfonso Garcia

            Perdon CC, creo que no te he formulado bien la pregunta.

            Tu argumento para decir que la masa del foton es cero es el siguiente: una particula que vaya a la velocidad de la luz tendra un s^2=0 y como se encuentra sobre el cono de luz, cualquier cuadrivector que construyamos tendra como modulo cero. Por tanto el modulo del cuadrimomento es cero.

            Ahora viene mi pregunta. Por que para particulas que se encuentran en el cono de luz (p.e. el foton) todos los cuadrivectores que cinstruyamos tienen que valer lo mismo (es decir, cero)?

            Por eso antes te he preguntado para una particula con masa. Porque en ese caso esta claro que s^2 es mayor que cero pero no tiene porque valer como p^2 (que es siempre igual a m^2).

            Es decir, para particulas sobre el cono de luz todos los cuadrivectores valen igual pero para particulas dentro del cono eso ya no es obligatorio. Por que?

            Espero que esta vez haya quedado mas clara la pregunta. Perdon por mi torpeza

            • Por que para particulas que se encuentran en el cono de luz (p.e. el foton) todos los cuadrivectores que cinstruyamos tienen que valer lo mismo (es decir, cero)?

              Esto se puede responder en varios niveles. Lo que define a cualquier vector que está en el cono de luz es justamente que su módulo es 0. Esto además se puede justificar estudiando la estructura algebráica, representaciones, del grupo de Poincaré, pero es demasiado elevado para explicarlo en esta entrada.

              Es decir, para particulas sobre el cono de luz todos los cuadrivectores valen igual pero para particulas dentro del cono eso ya no es obligatorio. Por que?

              Pues porque las partículas dentro del cono pueden acelerar, frenar, cambiar de dirección, etc. Es decir, pueden estar sometidas a fuerzas/interacciones y entonces tienen una dependencia con su inercia, es decir, su masa. Por supuesto, esto también sale de estudiar las representaciones (temporales) del grupo de Poincaré.

              Espero haberte aclarado esta duda que por otra parte es muy interesante.

              • Ok. Ha salido Poincare a relucir (era de esperar, XD). Si una propiedad del grupo de Poincare es que todos los cuadrivectores que construyas sobre el cono de luz tienen modulo cero, eso va a misa, XD. La verdad es que tendre que repasar el grupito porque esta cosas siempre es bonito acordarte.

                Te he hecho esta pregunta para probarte un poco, jeje. Pero la verdad es que no dudaba que supieras responderme con la facilidad que haces siempre.

                Sabes lo que pasa? que si miras cualquier introduccion sobre Relatividad Especial la mayoria te demuestran que la masa del foton es cero de la siguiente forma:

                1) El momento en SR se define como: p_i=m·\gamma·v_i

                2) La energia como: E=m·\gamma·c

                3) Si construimos un cuadrimomento de la forma p_\mu=(E/c,p_i) entonces su modulo valdra p_\mu·p^\mu=m^2*c^4. Ademas, este modulo es invariante bajo transformaciones Lorentz. Por tanto la masa la misma para cualquier observador.

                4) (Ahora viene el truco del almendruco) Los fotones tienen E=pc por tanto si sustitumos en el cuadrimomento esta E entonces nos sale que el modulo del cuadrimento es cero. Por tanto, la masa del foton es cero.

                Pero toman como principio que la energia del foton es E=pc. Claro, asi yo tambien demuestro que la masa es cero!

                La clave esta en decir porque E=pc. Por eso tu forma de demostralo es la correcta. Primero partimos de particulas con v=c, por tanto su s^2=0. Ademas cualquier cuadrivector que construyamos con la condicion s^2=0 tambien tendra modulo cero (bendito Poincare). Entonces el cuadrimomento es cero y por tant la masa es cero.

                Bueno, siento la parrafada pero escribiendolo me entero mejor. En cualquier caso, enhorabuena por tu magnifico blog porque en cada entrada nos demuestras (tanto a los que somos fisicos como los que no) que para divulgar no hace falta inventarse cosas raras, simplemente hay que saber muuuuucho de lo que se habla y tener muuuucha paciencia con gente como yo, XD.

                • Nada hombre, comentarios así son los que dan sentido a las entradas y los que enriquecen el blog. Ojalá pasara en todas las entradas.

                  Cuando se escribe hay muchas cosas que las ves «fáciles» y no las dices, o demasiado complicadas y tampoco las dices.

                  En este caso, estoy de acuerdo contigo, muchas veces no se demuestra que el fotón no tiene masa, sino que se asume (evidentemente porque hay otras pruebas que lo apoyan) y entonces hacen la deducción que comentabas. Pero tienes razón, es un poco had hoc, por eso he intentado mostrar que los vectores nulos tienen módulo 0 con la métrica de Minkowski, lo que no había dicho es justamente que entonces todos los vectores nulos tienen esa propiedad. Y evidentemente el cuadrimomento de una partícula que se mueve a c tiene que ser un vector nulo.

                  En realidad, como se ve eso bien es estudiando las propiedades del casimir de Poincaré, pero eso es para mayores 🙂

                  Algún día nos meteremos en ello, de hecho la entrada está como borrador desde casi el principio del blog, algún día la terminaré y la publicaré (la leeremos 3 :P).

                  Muchas gracias por tu explicación, creo que ha aclarado mucho la entrada.

  52. ¿Por qué nos pone tan nerviosos, intuir que algo «no tiene masa»?
    ¿Es nuestro más preciado bien?
    ¿Es por una cuestión teórica-formal?
    No veo el problema, es solo una cuestión, de serlo, de re-definir el concepto de masa y todo seria lo mismo, pero a la vez absolutamente maravilloso y fantástico.

    • De todos modos toda la teoría esta correlacionada y eso crea un círculo vicioso de confirmaciones encerradas en sus propios postulados base, de los que no se puede salir más que con algún experimento que cree las contradicciones necesarias. Lo mismo que ocurría con la mecánica; antes de la relatividad o con la electrodinámica antes de la quántica. Un paradigma, después del escrutinio histórico a que es sometido, después de ser redondeado; solo puede confirmar sus postulados y no sirve para dar un salto científico hacia otro paradigma. Se vuelve tautológico; cualquier intento de extraer información que este más allá de las bases experimentales que le dieron origen y delimitan sus alcances. Algo así como lo que no entienden los que tratan de fundir quántica con relatividad o que una particularice a la otra; y a la vez no destruir sus postulados base, la esencia que le da razón de ser,… Aun así es muy sospechoso que exista una especie de interacción entre los fotones y la gravedad;… y otras cosas más que pueden ser modeladas y descritas con precisión, desde otras perspectivas, que no expongo para no llamar la atención.
      Bueno algo en español siempre es bien recibido por nuestros amigos y los científicos del mañana, que hoy son niños. Ellos serán los encargados de dilucidar estas cosas y para los que esencialmente expongo mis meditaciones sin respuestas en estos blog,…

      Haz clic para acceder a articulojq.pdf

    • Yo desde que me enteré que la masa no era constante y variaba con la velocidad… y no sólo eso sino que era diferente según la velocaidad relativa al observador, ya nada me sorprende. Tiene energía y no masa? pues bien… la energía vendrá de otro lado, como del campo electromagnético.

  53. Pingback: Ondas em | Annotary

  54. Pingback: El fotón y la masa

  55. Cuando te refieres que el electron tiene momento, te refieres a que tiene cantidad de movimiento. Para mi el momento es otra cosa.

  56. Estoy con Begoña…Para mí es de una belleza general 😉

  57. Buen post! Gracias!

  58. Pingback: El fotón y la masa | Universo y F&iacute...

  59. Me encanta la calabaza. Bueno y el resto del post, estás que te sales estos días.

  60. Me encanta tu blog….
    Muchas gracias por compartir esto tan ….para mi es de una belleza especial…

    Seguire …

    • Muchas gracias a ti por leernos. 🙂

      • Hay una forma de explicar que es la masa que usa conceptos mecánicos simples. Como dice la ecuación m = P/c la masa es una cantidad de impulso dividido por c. La ecuación P = mc nos está diciendo cuanta cantidad de impulso constituye o contiene la masa, es decir, 1c Ns por kilogramo de masa. ¿puede una masa en reposo tener impulso? Parece que no pues P = mv y v es cero para la masa en reposo. Sin embrago la ecuación está ahí insitiendo en lo mismo.
        De modo que se lo podemos creer, para ello debemos suponer que la masa es una especie de aglutinado de impulsos que permanecen tirando tenso en todas direcciones y sentidos, lo cual hace que el sistema permanezca sin movimiento. Si aportamos ahora una cantidad de impulso en una determinada dirección estaremos desequilibrando el sitema el cual ahora habrá incrementado su masa de impulso, o sea su masa, y además estará en movimiento.
        Lo cierto es que partiendo de esta idea se puede explicar toda la relatividad especial, la masa y la inercia, es decir, las cuentas cuadran perfectamente, haciendo de la masa un almacen de impulsos y de la relatividad especial una forma de gestionar las entradas y salidas de impulsos de dicho almacén.

        • Hola Domingo. Aunque ya tiene algún tiempo tu comentario, me gustaría decirte que es muy interesante lo que planteas aquí:

          “debemos suponer que la masa es una especie de aglutinado de impulsos que permanecen tirando tenso en todas direcciones y sentidos, lo cual hace que el sistema permanezca sin movimiento. Si aportamos ahora una cantidad de impulso en una determinada dirección estaremos desequilibrando el sistema el cual ahora habrá incrementado su masa de impulso, o sea su masa, y además estará en movimiento”.

          De hecho mi teoría establece algo muy parecido. Aunque a esa “especie de aglutinado de impulsos que permanecen tirando tenso en todas direcciones y sentido”, yo lo llamo Campo. El mismo campo por cierto, que conocemos con el nombre de Campo Unificado y a través del cual se genera toda interacción entre partículas y objetos gravitacionales. Las cuales se designan con el nombre de débil, fuerte, electromagnética y gravitatoria, como ya sabes. Ahora bien, existe una relación estrecha entre masa y espacio-tiempo que mucha gente desconoce. Ya que todo objeto que ocupa espacio tiene masa.
          Esos impulsos que tu llamas, se generan por medio de la interacción que se produce entre las partículas que componen la materia con dicho campo. De manera que efectivamente, un sistema permanece sin movimiento porque las interacciones que se producen en dicho sistema están en equilibrio con el campo. En el caso de un átomo, al estar compuesto por partículas menores, que a su vez interaccionan con dicho campo, se genera una interacción con el mismo que actúa hacia dentro y hacia afuera del mismo. Con lo cual el átomo permanece en equilibrio y por tanto no experimenta movimiento. (Esto es válido para cualquier partícula con masa u objeto) Sin embargo el fotón es un caso especial. El fotón no ocupa espacio, ya que no está compuesto por partículas menores; y la mejor forma de demostrar esto es precisamente porque el campo, al no poder actuar en su interior, lo que hace es tirar hacia afuera del mismo y acelerarlo hasta la velocidad de la luz. Por eso el fotón no puede tener masa, ya que no ocupa espacio.
          Un cordial saludo.

          Este comentario está duplicado. Por favor ruego al administrador que borre el que aparece más arriba.

      • Creo que cometiste un error en esta parte, ya que creo que en la segunda línea de ecuaciones te sacas un c^2 de la nada.
        Para partículas que se mueven a velocidades inferiores a la velocidad de la luz lo que obtenemos es:
        (E/c)^2-p^2=m^2c^2
        E^2=p^2c^2+m^2c^4
        ¿Lo correcto no sería lo siguiente?:
        E^2=p^2c^2+m^2c^2
        O en todo caso lo siguiente:
        E^2=p^2+m^2c^4
        Espero tu respuesta, gracias.

        • El cálculo está bien como está. Has el análisis dimensional y te darás cuenta.

        • Manuel Sánchez Carrilero

          No te calientes más la cabeza si te lees mis comentarios anteriores llegarás a la siguiente expresión:
          E^2 = (mo^2)*(c^4) + (p^2)*(c^2)
          con
          E=mc^2
          p=mv
          m=mo/SQR(1-(v/c)^2)
          siendo mo la masa en reposo
          Si se trata de una partícula con masa en reposo nula (por ejemplo el fotón), la expresión de arriba dará
          E=p*c=mc*c=mc^2
          que tiene un significado de iteración del momento del fotón como tal partícula de masa m; aparte de la equivalencia energía masa.

Deja un comentario